quiz human sexuality

73
Human Sexuality 14Mar2009 DO NOT DISTRIBUTE - 1 - 1 – What is Human Sexuality? 1) Which of the following describes human sexuality? a) How men and women reproduce to further the human species b) The ways in which pleasure is derived from genital stimulation c) The methods of communication and expression of sexual feelings d) The ways in which humans are physically enabled to reproduce sexually e) The ways in which we experience and express ourselves as sexual beings Match the person with their scientific research: 2.1) Encyclopedia of sexuality between 1897 and 1910 a) Sigmund Freud 2.2) Sadomasochism, bestiality, necrophilia b) Havenlock Ellis 2.3) Sex drive is the principle motivating force c) Alfred Kinsey 2.4) Large scale sexual studies in 1930s and 1940s d) Richard von Krafft-Ebing 3) Which of the following focuses of biological sexual research would be the least applicable to patient care? a) Overcoming sexual problems b) Helping infertile couples conceive c) Hormones of sexual response d) Sexual arousal and response e) Drugs with sexual side-effects 4.1) Gender stereotypes are not inherent in our genetic heritage, but are acquired through cultural expectations and socialization. Which of the following is NOT a typical male role stereotype? a) Dependent b) Active c) Strong d) Logical e) Hunter 4.2) Which of the following is NOT true regarding cross-cultural variations? a) Kissing is common in most cultures b) In most cultures, sex is more frequent among young people c) Childhood masturbation is encouraged in most cultures d) Monogamy is the most prevalent form of relationship worldwide e) Polygyny is the most common form of worldwide polygamy 4.3) Which of the following describes fraternal polyandry? a) Fraternal male twins sharing multiple female partners b) Fraternal female twins sharing multiple male partners c) A male with two fraternal female partners d) A female with two fraternal male partners e) Fraternal male twins sharing fraternal female twins 5.1) Who developed the original Psychoanalytical Theory of human sexual behavior? a) Sigmund Freud b) Havenlock Ellis c) Alfred Kinsey d) Richard von Krafft-Ebing e) Watson and Skinner

Upload: medshare

Post on 12-Nov-2014

1.084 views

Category:

Documents


3 download

DESCRIPTION

Quiz Human Sexuality

TRANSCRIPT

Page 1: Quiz Human Sexuality

Human Sexuality 14Mar2009

DO NOT DISTRIBUTE - 1 -

1 – What is Human Sexuality?

1) Which of the following describes human sexuality?

a) How men and women reproduce to further the human species

b) The ways in which pleasure is derived from genital stimulation

c) The methods of communication and expression of sexual feelings

d) The ways in which humans are physically enabled to reproduce sexually

e) The ways in which we experience and express ourselves as sexual beings

Match the person with their scientific research:

2.1) Encyclopedia of sexuality between 1897 and 1910 a) Sigmund Freud

2.2) Sadomasochism, bestiality, necrophilia b) Havenlock Ellis

2.3) Sex drive is the principle motivating force c) Alfred Kinsey

2.4) Large scale sexual studies in 1930s and 1940s d) Richard von Krafft-Ebing

3) Which of the following focuses of biological sexual research would be the least

applicable to patient care?

a) Overcoming sexual problems

b) Helping infertile couples conceive

c) Hormones of sexual response

d) Sexual arousal and response

e) Drugs with sexual side-effects

4.1) Gender stereotypes are not inherent in our genetic heritage, but are acquired through

cultural expectations and socialization. Which of the following is NOT a typical male

role stereotype?

a) Dependent

b) Active

c) Strong

d) Logical

e) Hunter

4.2) Which of the following is NOT true regarding cross-cultural variations?

a) Kissing is common in most cultures

b) In most cultures, sex is more frequent among young people

c) Childhood masturbation is encouraged in most cultures

d) Monogamy is the most prevalent form of relationship worldwide

e) Polygyny is the most common form of worldwide polygamy

4.3) Which of the following describes fraternal polyandry?

a) Fraternal male twins sharing multiple female partners

b) Fraternal female twins sharing multiple male partners

c) A male with two fraternal female partners

d) A female with two fraternal male partners

e) Fraternal male twins sharing fraternal female twins

5.1) Who developed the original Psychoanalytical Theory of human sexual behavior?

a) Sigmund Freud

b) Havenlock Ellis

c) Alfred Kinsey

d) Richard von Krafft-Ebing

e) Watson and Skinner

Page 2: Quiz Human Sexuality

Human Sexuality 14Mar2009

DO NOT DISTRIBUTE - 2 -

5.2) The Psychoanalytical Theory said the Ego was meant to shield the conscious mind

using defense mechanisms, such as forgetting traumatic events. Although sexual ideas

may be banished to unconsciousness, how may they be manifested?

a) Dreams

b) Aggression

c) Promiscuity

d) Fear

e) All of the above

5.3) Who developed the concept of erogenous zones, stating that many parts of the body

(not just genitals) are responsive to sexual stimulation?

a) Sigmund Freud

b) Havenlock Ellis

c) Alfred Kinsey

d) Richard von Krafft-Ebing

e) Watson and Skinner

5.4) The Oedipus Complex is a conflict during which stage of infant development where

the boy wishes to possess his mother sexually and perceives his father as a rival in love?

a) Oral

b) Anal

c) Phallic

d) Latency

e) Genital

5.5) Behaviorists Watson and Skinner speculated that punishing children for sexual

exploration (e.g. masturbation) would have what future results?

a) Associating sexual thoughts with feelings of guilt or anxiety

b) Associating sexual stimulation with feelings of guilt or anxiety

c) A desire to become involved dominating sexual relationships

d) A desire to seeks sexual pleasure through any means possible

e) Associating sexual acts with childhood

5.6) In the social-learning theory of human sexuality, what term is used for acquiring

knowledge and skills by observing others?

a) Plagiarizing

b) Duplicating

c) Replicating

d) Modeling

e) Copulation

6) Which of the following contributes the least to differences in sociocultural

perspectives of sexuality?

a) Age

b) Education

c) Religion

d) Ethnicity

e) Gender

7) It is important to include many perspectives in the understanding of sexuality as what

is meaningful, right, wrong, and natural vary widely between different cultures.

a) True

Page 3: Quiz Human Sexuality

Human Sexuality 14Mar2009

DO NOT DISTRIBUTE - 3 -

b) False

8.1) Knowing enough about the biology of sex will enable someone to make the right

sexual decisions.

a) True

b) False

8.2) Some ancient Greek men took on an adolescent male as a lover and pupil.

a) True

b) False

8.3) In general, women of the ancient world were treated as personal property (chattels).

a) True

b) False

8.4) The production of illustrated sex manuals originated in modern times.

a) True

b) False

8.5) Graham crackers originated as a means for helping young men control their sexual

appetites.

a) True

b) False

8.6) The Trobriander tribe in Africa expected young boys and girls to engage in

intercourse when they first became biologically old enough.

a) True

b) False

8.7) The female redback spider eats the male redback spider immediately after sex.

a) True

b) False

8.8) Psychoanalysts may believe dreams of airplanes, bullets, snakes, sticks, and similar

objects symbolize male genitalia (phallic symbolism). Research has shown significant

evidence of this being true.

a) True

b) False

9.1) Sexual relations between a person and an animal is called:

a) Incest

b) Monogamy

c) Pederasty

d) Bestiality

e) Polygyny

9.2) Which of the following is defined as sexual intercourse between two people who are

not married?

a) Coitus

b) Copulation

c) Fornication

d) Monogamy

e) Polygamy

9.3) Incest is defined as sex between a person and:

a) An animal

b) A plant

Page 4: Quiz Human Sexuality

Human Sexuality 14Mar2009

DO NOT DISTRIBUTE - 4 -

c) A close relative

d) A lover

e) A spouse

9.4) Achieving sexual gratification through hurting or humiliating others is called:

a) Medical school

b) Pederasty

c) Bisexual

d) Masochism

e) Sadism

9.5) Pederasty is a sexual love of:

a) Women

b) Men

c) Girls

d) Boys

e) Animals

9.6) Cunnilingus and fellatio are the acts of sexual activity involving the female and male

genitalia, respectively.

a) True

b) False

2 – Research Methods in Human Sexuality

1) Which of the following is NOT a goal of the science of human sexuality?

a) Formulating a research question

b) Framing the research question in the form of a hypothesis

c) Testing the hypothesis

d) Drawing conclusions

e) Using different species to confirm results

2.1) During Kinsey’s survey study between 1938 and 1949, a correlation between which

of the following was found?

a) Age and masturbation

b) Race and duration of intercourse

c) Education and participation in oral sex

d) Wealth and frequency of anal sex

e) Height and number of sexual partners

2.2) The results of Kinsey’s survey showed people seemed to find it difficult to describe

which of the following?

a) If they had participated in sexual activities

b) Which sexual activities they did the most

c) Which sexual positions they had done

d) The frequency of sexual activities

e) The number of sexual partners they had

3) Masters and Johnson were the first to conduct what type of sexual behavior study?

a) Dating in a public setting

b) Sexual stimulation in a laboratory setting

c) Sexual stimulation in a private setting

d) Masturbation in a public setting

Page 5: Quiz Human Sexuality

Human Sexuality 14Mar2009

DO NOT DISTRIBUTE - 5 -

e) Interracial sexual contact in a public setting

4.1) A sexual study is being conducted in which participants are free to withdraw from

the study at any time and without penalty. Which of the following ethical research issues

does this address?

a) Exposing participants to harm

b) Confidentiality

c) Informed consent

d) Use of deception

4.2) A sexual study is being conducted in which applicants must show identification to

prove they are adults prior to becoming participants in the study. Which of the following

ethical research issues does this address?

a) Exposing participants to harm

b) Confidentiality

c) Informed consent

d) Use of deception

5.1) Some sexual researchers have engaged in “swinging” with the people they studied.

a) True

b) False

5.2) Masters and Johnson created a transparent artificial penis containing photographic

equipment to study female sexual response.

a) True

b) False

5.3) People who attend church regularly tend to be more satisfied with their relationships.

a) True

b) False

5.4) Studies have proven that Sildenafil (Viagra) causes risky sexual behavior.

a) True

b) False

5.5) There is evidence that the victims of genital mutilation (the women what are

circumcised) tend to internalize the beliefs of their culture. Not only do they generally

support female circumcision, they also accept the right of husbands to beat their wives.

a) True

b) False

5.6) Researchers publish the names of participants of sex research in professional

journals.

a) True

b) False

6.1) Anthropomorphism is defined as attributing human characteristics to:

a) Sexual objects

b) Plants

c) Animals

d) Liquids

e) Automobiles

6.2) Which of the following describes what a penile strain gauge measures?

a) How much weight the penis can hold

b) The length of the penis during sexual arousal

Page 6: Quiz Human Sexuality

Human Sexuality 14Mar2009

DO NOT DISTRIBUTE - 6 -

c) The circumference of the penis during sexual arousal

d) The length of the penis without sexual arousal

e) The circumference of the penis without sexual arousal

6.3) A vaginal plethysmograph is a tampon-shaped probed that is inserted into the vagina,

using light reflected from the vaginal walls to measure which of the following?

a) Lymphatic flow

b) Orgasm duration

c) Orgasm strength

d) Vasocongestion

e) Anatomical relations

3 – Female Sexual Anatomy and Physiology

1.1) Which part of the female sexual anatomy consists of fatty tissue covering the pubic

bone joint to serve as a cushion during sex?

a) Mons veneris

b) Labia majora

c) Labia minora

d) Introitus

e) Clitoris

1.2) The vestibule is the area within the labia minora that contains the vaginal opening

and which of the following?

a) Clitoris

b) Labia majora

c) Mons veneris

d) Introitus

e) Urethra

1.3) Anatomical changes to which of the following areas is used in arguments of virginity

and in arguments of sexual abuse, although such evidence is controversial?

a) Clitoris

b) Labia majora

c) Mons veneris

d) Introitus

e) Urethra

2) Which of the following structures does NOT underlie the vulva?

a) Vestibular bulbs

b) Bartholin glands

c) Prepuce

d) Vaginal sphincters

e) Anal sphincters

3) Which of the following is the only female sex organ used specifically for pleasure and

contains corpora cavernosa?

a) Mons veneris

b) Hymen

c) Labia minora

d) Introitus

e) Clitoris

Page 7: Quiz Human Sexuality

Human Sexuality 14Mar2009

DO NOT DISTRIBUTE - 7 -

4.1) Female genital cutting (FGC), also known as female genital mutilation (FGM) or

female circumcision, refers to "all procedures involving partial or total removal of the

external female genitalia or other injury to the female genital organs whether for cultural,

religious or other non-therapeutic reasons." The World Health Organization defines

infibulation (Type III FGC) as which of the following?

a) Removal of the clitoris (clitoridectomy)

b) Removal of the labia minora

c) Surgical closure of the labia majora

d) All of the above

4.2) Infibulation is the most common in which of the following African areas?

a) Somalia, Egypt, Sudan

b) Libya, Algeria, Morocco

c) Niger, Chad, Nigeria

d) Angola, Zambia, Namibia

e) Madagascar, Mozambique, South Africa

4.3) Which of the following is NOT a common complication of FGC/FGM procedures?

a) Future sexual dysfunction

b) Infertility

c) Bleeding

d) Edema

e) Infection

5) The condition of the hymen can be used to prove virginity; that is, an intact hymen

means the female has not had penetrating sex.

a) True

b) False

6.1) The opening in the middle of the cervix leading to the uterus is called the:

a) Os

b) Hole

c) Slit

d) Pouch

e) Introitus

6.2) Which of the following is the upper end of the vagina whose secretions contribute to

the chemical balance of the vagina?

a) Ovaries

b) Uterus

c) Fallopian tubes

d) Cervix

e) Bartholin glands

7.1) Which of the following pathologies is characterized by menstrual pain?

a) Vaginitis

b) Cervical cancer

c) Endometriosis

d) Ectopic pregnancy

e) Ovarian cyst

7.2) Which of the following pathologies can be most easily prevented with proper

hygiene?

Page 8: Quiz Human Sexuality

Human Sexuality 14Mar2009

DO NOT DISTRIBUTE - 8 -

a) Vaginitis

b) Cervical cancer

c) Endometriosis

d) Ectopic pregnancy

e) Ovarian cyst

8) Which of the following is NOT part of the routine pelvic exam?

a) Inspection of the external genitalia for swelling and irritation

b) Inspection of the internal genitalia with a speculum

c) Bimanual exam with two fingers in the vagina and other hand on the abdomen

d) Rectovaginal inspection with a finger in the vagina and a finger in the anus

e) Prostate exam with two fingers in the anus

9) Menopause is the cessation of menstruation that generally takes place in women ages

46-50. The associated falling off of production of hormones causes all of the following

menopause symptoms EXCEPT:

a) Hot flashes

b) Joint pain

c) Night sweats

d) Cold sweats

e) Weight gain

10) Although there is a social-religious stigma behind coitus during menstruation,

contraction of the uterus during orgasm may help relieve cramping and helps to clear the

lining and clotted blood left inside the uterus.

a) True

b) False

11.1) Dysmenorrhea (pain during menstruation) is very common in women. Which of the

following is NOT a symptom associated with menstruation?

a) Bloating

b) Nausea

c) Headaches

d) Fever

e) Cramps

11.2) Elevated levels of prostaglandins during menstruation is associated with which of

the following symptoms?

a) Bloating

b) Nausea

c) Headaches

d) Fever

e) Cramps

11.3) Psychologically, women need recognize that there is nothing wrong with them

during dysmenorrhea, this a naturally occurring event and they are blameless for their

physical discomfort.

a) True

b) False

12.1) Pudendum is derived from Latin meaning “something to be ashamed of.”

a) True

b) False

Page 9: Quiz Human Sexuality

Human Sexuality 14Mar2009

DO NOT DISTRIBUTE - 9 -

12.2) Many males (and even some females) believe erroneously that for women urination

and coitus occur through the same bodily opening.

a) True

b) False

12.3) Breast size helps determine the quantity of milk that can be produced.

a) True

b) False

12.4) Abortions and silicone breast implants have been shown to increase a woman’s risk

of breast cancer.

a) True

b) False

12.5) Monthly breast self-examinations (BSEs) are recommended by the American

Cancer Society.

a) True

b) False

12.6) Many women have hot flashes at menopause.

a) True

b) False

13.1) The circular area of colored skin surrounding the nipple is called the:

a) Areola

b) Bartholin gland

c) Crura

d) Os

e) Prepuce

13.2) An operation to remove the uterus but not the cervical stump is called a(n):

a) Amenorrhea

b) Episiotomy

c) Hysterectomy

d) Lumpectomy

e) Mastectomy

13.3) Climacteric is a long-term process synonymous with menopause, or a phase in a

man’s life corresponding to female menopause.

a) True

b) False

13.4) Anorexia nervosa is an eating disorder characterized by spells of excessive eating.

a) True

b) False

13.5) The clitoral crura are an internal portion of the clitoris that are shaped like an

inverted "V" with the vertex of the "V" connecting to the clitoral body. They are near the

vestibular bulbs, also known as the clitoral bulbs.

a) True

b) False

13.6) Surgical removal of the entire breast is known as a(n):

a) Amenorrhea

b) Episiotomy

c) Hysterectomy

Page 10: Quiz Human Sexuality

Human Sexuality 14Mar2009

DO NOT DISTRIBUTE - 10 -

d) Lumpectomy

e) Mastectomy

13.7) A fibroadenoma is a:

a) Saclike structure filled with fluid or diseased material

b) A tumor that is lethal or likely to cause harm

c) Pain in the breast

d) A benign, fibrous tumor

e) Surgical removal of a lump from the breast

13.8) A surgical incision in the perineum that may be made during childbirth to protect

the vagina from tearing (although the prevalence of this surgery is declining) is known as:

a) Amenorrhea

b) Episiotomy

c) Hysterectomy

d) Lumpectomy

e) Mastectomy

13.9) The internal female genitals are known as the pudendum.

a) True

b) False

13.10) The prepuce is the fold of skin covering the glans of the clitoris and is also known

as the clitoral hood.

a) True

b) False

13.11) Derived from the French term “shower”, this device is meant to clean the vagina

but has been associated with long term complications such as yeast infections.

a) Dildo

b) Speculum

c) Pap test

d) Tampon

e) Douche

13.12) Cramps or painful menstruation associated with a disease process is known as:

a) Primary amenorrhea

b) Primary dysmenorrhea

c) Secondary amenorrhea

d) Secondary dysmenorrhea

e) Menarche

13.13) Which of the following is often caused by hormonal disturbances from the

hypothalamus (body temperature, motivation, emotion) and the pituitary gland (prolactin,

oxytocin, growth hormone) or from premature menopause, or intrauterine scar formation.

a) Primary amenorrhea

b) Primary dysmenorrhea

c) Secondary amenorrhea

d) Secondary dysmenorrhea

e) Menarche

13.14) Growth and inflammation outside of the inner membrane of the uterus is:

a) Cystitis

b) Endometriosis

Page 11: Quiz Human Sexuality

Human Sexuality 14Mar2009

DO NOT DISTRIBUTE - 11 -

c) Osteoporosis

d) Vaginitis

e) Mittelschmerz

13.15) Traits that distinguish women from men but are not directly involved in

reproduction are known as secondary sex characteristics.

a) True

b) False

13.16) The first menstration is known as:

a) Primary amenorrhea

b) Primary dysmenorrhea

c) Secondary amenorrhea

d) Secondary dysmenorrhea

e) Menarche

13.17) Ovulation pain is known as:

a) Cystitis

b) Endometriosis

c) Osteoporosis

d) Vaginitis

e) Mittelschmerz

13.18) The Papanikolaou test (Pap test) is a medical screening method primarily designed

to detect premalignant and malignant processes in the ectocervix.

a) True

b) False

Match the definition with the term:

13.19) Hypothalamus hormone that causes pituitary gonadotropin release a) Prolactin

13.20) Stimulates the production of milk b) Oxytocin

13.21) Stimulates follicle development c) Follicle-stimulating hormone (FSH)

13.22) Stimulates uterine contractions and milk ejection d) Luteinizing hormone (LH)

13.23) Triggers ovulation e) Gonadotropin-releasing hormone (Gn-RH)

13.24) Estrus refers to the phase in the estrous cycle when the woman is sexually

receptive (e.g. “in heat”) and may have associated traits such as the lordotic reflex.

a) True

b) False

13.25) The first phase of the menstrual cycle that lasts about nine or ten days is known as:

a) Ovulation

b) Pituitary

c) Premenstrual syndrome (PMS)

d) Proliferation

e) Secretory

13.26) A premenstrual phase characterized by severe changes in mood and impairment of

functions at work, at school, or in social relationships is known as:

a) Premenstrual syndrome (PMS)

b) Premenstrual dysphoric disorder (PMDD)

c) Anorexia nervosa

d) Mastalgia

e) Hysteria

Page 12: Quiz Human Sexuality

Human Sexuality 14Mar2009

DO NOT DISTRIBUTE - 12 -

14.1) Which positions is NOT a recommended position for a breast self exam (BSE)?

a) In the shower

b) Standing in front of a mirror

c) Lying down

d) During sexual activity

14.2) Which of the following is NOT true regarding menopause?

a) Menopause is normal

b) Menopause is not considered a disease

c) After menopause, women need complete replacement of estrogen

d) Menopause is not always accompanied with depression and anxiety

e) Not all women experience hot flashes at menopause

f) A woman who has had a hysterectomy may still undergo menopause

g) Menopause does not end a woman’s sexual appetite

h) A woman’s general level may be higher after menopause

4 – Male Sexual Anatomy and Physiology

1.1) Which of the following areas of the penis would be the least sensitive?

a) Glans

b) Corona

c) Frenulum

d) Crura

1.2) The penis contains which of the following types of cylinders?

a) 1 Corpus cavernosum, 1 Corpus spongiusum

b) 1 Corpus cavernosum, 2 Corpora spongiosa

c) 2 Corpora cavernosa, 1 Corpus spongiosum

d) 2 Corpora cavernosa, 2 Corpora spongiosa

e) 3 Corpora cavernosa, 3 Corpora spongiosa

2) Which of the following is NOT true regarding male circumcision?

a) It involves the removal of the prepuce

b) Some believe circumcision improves hygiene and removes smegma (a cheese-

like, foul smelling secretion that may accumulate)

c) Circumcision is least common amongst European American men

d) Research has shown circumcision can reduce the spreading of HIV

e) Circumcision can lessen sexual sensation

f) The American Academy of Pediatrics does not recommend circumcision as a

universal procedure to prevent sexually transmitted disease

3.1) Zilbergeld (1999) reported that women complained about their partner’s penis size

more than other relationship issues such as communication or emotional atmosphere.

a) True

b) False

3.2) Which of the following would NOT reduce penis size?

a) Cold air

b) Cold water

c) Fear

d) Anxiety

e) Sexual excitement

Page 13: Quiz Human Sexuality

Human Sexuality 14Mar2009

DO NOT DISTRIBUTE - 13 -

4) Which of the following raises or lowers the testicles in the scrotum in an attempt to

keep testicular temperature about 5 to 6 degrees below body temperature (98.6ºF)?

a) Dartos muscle

b) Spermatic cord

c) Vas deferens

d) Cremaster muscle

e) Corpus cavernosa

5.1) Which of the following is the correct route of sperm?

a) Seminiferous tubules, epididymis, Vas deferens, ejaculatory duct, urethra

b) Seminiferous tubules, Vas deferens, epididymis, ejaculatory duct, urethra

c) Seminiferous tubules, Vas deferens, ejaculatory duct, epididymis, urethra

d) Seminiferous tubules, ejaculatory duct, urethra, epididymis, Vas deferens

e) Epididymis, Seminiferous tubules, Vas deferens, ejaculatory duct, urethra

5.2) The fluid produced by which of the following is rich in fructose, which nourishes the

sperm, and secretes 70% of the accessory fluid for ejaculation?

a) Prostate

b) Bulbourethral gland

c) Seminal vesicle

d) Epididymis

e) Seminiferous tubules

6) Cancer of the testicles is a rare form of cancer (1% of all new cancers in men) and

between the ages of 20-34 accounts for what percentage of cancer deaths?

a) 0.5%

b) 1%

c) 2%

d) 5%

e) 10%

7) Testicular self-exams should be done regularly by rolling each testicle between the

thumb and fingers. Which of the following is NOT a warning sign during self-exam per

the National Cancer Institute?

a) Slight enlargement of one of the testes

b) Change in consistency of the testicle

c) Dull aches in the lower abdomen or groin

d) Sensation of dragging or heaviness in the testicles

e) Burning sensation during urination

8.1) What percentage of men over the age of 50 will develop benign prostatic hyperplasia

(BPH), which can be treated with alpha blockers?

a) 10%

b) 20%

c) 50%

d) 70%

e) 80%

8.2) How many men will develop prostate cancer, which is life-threatening, in their life?

a) 1 in 8

b) 1 in 6

c) 1 in 4

Page 14: Quiz Human Sexuality

Human Sexuality 14Mar2009

DO NOT DISTRIBUTE - 14 -

d) 1 in 2

9.1) Men have nocturnal erections every 90 minutes or so as they sleep, generally

occurring during rapid eye movement (REM).

a) True

b) False

9.2) Men must make a conscious effort to become aroused (erect) during sexual touching.

a) True

b) False

10) Because of a lack of nerve connection between genital organs and the brain, men with

spinal injuries can achieve erection sand ejaculate in response to direct tactile stimulation

of the penis even if they have no sensation.

a) True

b) False

11) What division of the nervous system is involved in male erection?

a) Somatic afferent system

b) Somatic efferent system

c) Autonomic (both branches)

d) Autonomic sympathetic system

e) Autonomic parasympathetic system

12.1) Peyronie disease is a disorder involving:

a) Overgrowth of the labia minora

b) Oversize clitoris

c) Overgrowth of the penile prepuce

d) Excessive curvature of the penis

e) An erection lasting for hours

12.2) Priapism is a condition described as:

a) Overgrowth of the labia minora

b) Oversize clitoris

c) Overgrowth of the penile prepuce

d) Excessive curvature of the penis

e) An erection lasting for hours

12.3) In retrograde ejaculation, semen is ejected into the bladder.

a) True

b) False

13.1) During the expulsion stage of ejaculation, which of the following is most involved

in movement of semen?

a) Prostate

b) Seminal vesicles

c) Ampulla of Vas Deferens

d) Muscles at the base of the penis

e) Urethral bulb

13.2) Which center(s) in the spinal cord regulate ejaculation?

a) Sacral

b) Lower lumber

c) Higher lumbar

d) Sacral and lower lumber

Page 15: Quiz Human Sexuality

Human Sexuality 14Mar2009

DO NOT DISTRIBUTE - 15 -

e) Sacral and higher lumber

14.1) The penis is made of bone and muscle.

a) True

b) False

14.2) The father determines a child’s gender.

a) True

b) False

14.3) Sperm of the fruit fly (drosophila) are longer than human sperm.

a) True

b) False

14.4) Morning erections indicate the need to urinate.

a) True

b) False

14.5) Men can will themselves to have an erection.

a) True

b) False

14.6) The penis has a mind of its own.

a) True

b) False

14.7) Men can orgasm without ejaculation.

a) True

b) False

15.1) A condition in which the foreskin is so tight it cannot be withdrawn from the glans

penis is known as:

a) Cryptorchidism

b) Circumcision

c) Phimosis

d) Paraplegia

e) Urethritis

15.2) What is secreted by interstitial (Leydig) cells?

a) Sperm

b) Estrogen

c) Testosterone

d) Alkalizing fluid for the ejaculate

e) Fructose for the ejaculate

15.3) Androgens are male sex hormones.

a) True

b) False, they are female sex hormones

15.4) After a vasectomy (ligation of Vas Deferens), how long should the patient wait

before having sex again?

a) A couple days

b) A couple weeks

c) Three months

d) Six months

e) One year

16.1) Prostate cancer progresses slowly and is usually detected by which means?

Page 16: Quiz Human Sexuality

Human Sexuality 14Mar2009

DO NOT DISTRIBUTE - 16 -

a) Digital rectal exam (DRE)

b) Prostate specific antigen (PSA)

c) CT scan

d) MRI scan

e) All of the above

16.2) What groups ae at increased risk for prostate cancer?

a) European American men with no family history

b) European American men with a family history

c) African American men with no family history

d) African American men with a family history

17) Koro-Genital Reaction Syndrome is a disorder found in some Asian men who believe

their penis will shrink and retract into their body even though it is not anatomically

possible, such as during attempts to urinate or when it becomes could outside.

a) True

b) False

18) Prescriptions for testosterone replacement therapy has shown conclusive results.

a) True

b) False

5 – Sexual Arousal and Response

1) When it comes to sexual arousal, people tend to be:

a) Taste oriented

b) Smell oriented

c) Sound oriented

d) Sight oriented

2) Smell may also indicate preference of sexual partners. In one study it was found that

gay men preferred the axillary odors of gay men over heterosexuals or lesbians. This

same study found that lesbians and heterosexuals preferred the axillary odors of

heterosexual men and lesbians to that of gay men.

a) True

b) False

3) Which of the following is NOT a primary erogenous zone?

a) Genitals

b) Outer thigh

c) Perineum

d) Anus

e) Breasts

4) Which of the following play a more minor role in sexual arousal?

a) Sight and touch

b) Hearing and sight

c) Hearing, taste, and smell

d) Touch and smell

e) Taste, sight, and touch

5.1) Any sexual arousal from non-drug aphrodisiacs is most likely a result of the placebo

effect as they have not been shown to be effective.

a) True

Page 17: Quiz Human Sexuality

Human Sexuality 14Mar2009

DO NOT DISTRIBUTE - 17 -

b) False

5.2) Which of the following is likely the most potent aphrodisiac?

a) L-dopa

b) Arginine

c) Amyl nitrate

d) Testosterone

e) Nitroglycerine

6) Which of the following drugs enhances sexual arousal in males and females?

a) Marijuana

b) Alcohol

c) Cocaine

d) Amphetamines

e) Hallucinogens

7) The cerebral cortex is involved in sexual ____ and the limbic system is involved in

sexual ____.

a) Arousal; Arousal

b) Arousal; Thoughts

c) Thoughts; Arousal

d) Thoughts; Thoughts

8) Which of the following hormones has been shown to be the dominating factor in the

sex drive of both men and women, and may be involved in organizing the fetal brain

toward a masculine or feminine direction.

a) Estrogen

b) Progesterone

c) Testosterone

d) Oxytocin

e) Leutinizing hormone

9) Which stage of sexual response (Masters and Johnson) is associated with

vasocongestion and increased lubrication?

a) Excitement phase

b) Plateau phase

c) Orgasmic phase

d) Resolution phase

10) Unlike the four-stage model of sexual response, the three-stage model by Kaplan

does not have a specific order and is based mainly on:

a) Orgasm

b) The man

c) The woman

d) Sexual dysfunction

e) The lunar calendar

11) Which of the following is true of multiple orgasms?

a) Men can have them and women cannot

b) Women can have them and men cannot

c) Men and women can have them

d) Neither men nor women can have them

Page 18: Quiz Human Sexuality

Human Sexuality 14Mar2009

DO NOT DISTRIBUTE - 18 -

12) Although Masters and Johnson felt there was one type of orgasm that men and

women had, Singer and Singer proposed there were:

a) No definite number of types

b) Two types

c) Three types

d) Four types

e) Five types

13) The Grafenberg spot (G-spot) is anatomically located 1 to 2 inches from the vaginal

entrance on what wall?

a) Lateral

b) Anterior

c) Posterior

d) Encircling

14.1) Menstrual cycles of women who live together tend to become synchronized.

a) True

b) False

14.2) The brain is considered an erogenous zone.

a) True

b) False

14.3) Spanish fly (cantharidin) inflames the urethra causing a burning sensation, which

can be misinterpreted as a sexual feeling.

a) True

b) False

14.4) In a study by Proctor (1974), written descriptions of male and female orgasms

could not be told apart.

a) True

b) False

14.5) Masters and Johnson (1966) found that orgasms during masturbation were more

physiologically intense than those experienced during intercourse.

a) True

b) False

15.1) Sex flush is a reddish rash that appears on what part of the body during the

excitement phase of the sexual response cycle?

a) Labia

b) Face

c) Chest

d) Hands

e) Buttocks

15.2) Sex skin refers to reddening of what part of the body during the plateau phase of the

sexual response cycle?

a) Labia

b) Face

c) Chest

d) Hands

e) Buttocks

Page 19: Quiz Human Sexuality

Human Sexuality 14Mar2009

DO NOT DISTRIBUTE - 19 -

16.1) Studies have shown the perspiration (e.g. underarm secretions) may make people

more sexually attractive.

a) True

b) False

16.2) Which of the following is considered a date-rape drug?

a) AZT

b) Metoprolol

c) HCTZ

d) Cialis

e) Rohypnol

6 – Gender Identity and Gender Roles

1.1) Which chromosome pair determines zygotic sex?

a) 13

b) 19

c) 21

d) 23

1.2) Each male (Wolffian) duct develops into:

a) Epididymis

b) Vas Deferens

c) Seminal vesicle

d) All of the above

2) Gender dysphoria refers to:

a) Pre-natal exposure to testosterone

b) Being born homosexual

c) Having homosexual tendencies

d) Elation with one’s gender

e) Incongruity between anatomic gender and gender identity

Match the following terms with their definition:

3.1) A person with a strong desire to be the opposite sex a) Hermaphroditism

3.2) Having gonads of one sex and genitalia of the opposite sex b) Intersexualism

3.3) Having ovarian and testicular tissue c) Transsexualism

3.4) Male-to-female surgical sex changes can be done more precisely than female-to-

male (phalloplasty) sex changes.

a) True

b) False

4.1) Without male sex hormones, female external genitalia would develop.

a) True

b) False

4.2) 17 out of 18 boys who appeared to have female external sex organs suddenly

developed male sex organs at puberty, when male sex hormones went to work. These

boys were diagnosed with Dominican Republic syndrome, a genetic enzyme disorder that

prevents testosterone form masculinizing the external genitals.

a) True

b) False

4.3) The sex of a crocodile depends on the temperature at which eggs develop.

Page 20: Quiz Human Sexuality

Human Sexuality 14Mar2009

DO NOT DISTRIBUTE - 20 -

a) True

b) False

4.4) Sex reassignment surgery can implant internal genitalia and internal reproductive

organs.

a) True

b) False

5.1) Autogynephilic refers to:

a) Females who are sexually stimulated by fantasies of their bodies being male

b) Females who are sexually stimulated by fantasies of their bodies

c) Males who are sexually stimulated by fantasies of their bodies being female

d) Males who are sexually stimulated by fantasies of their bodies

5.2) Congenital adrenal hyperplasia is the most common form of intersexualism and

involves:

a) Female internal genitalia and male external genitalia

b) No internal genitalia and male external genitalia

c) Male internal genitalia and female external genitalia

d) No internal genitalia and female external genitalia

5.3) A patient is born with karotype 46XY but has no problem with production of

testosterone or mullarian inhibiting hormone. Their vagina ends in a blind pouch and

testes are lodged in the inguinal canal. Which of the following is the best description of

this patient:

a) Male pseudo-hermaphrodite

b) Female pseudo-hermaphrodite

c) Androgen insensitivity syndrome

d) Turner’s syndrome

e) Klinefelter syndrome

5.4) A patient is born as a male with karotype 47XXY and does not develop appropriate

male secondary sex characteristic. Which of the following is the best description of this

patient:

a) Male pseudo-hermaphrodite

b) Female pseudo-hermaphrodite

c) Androgen insensitivity syndrome

d) Turner’s syndrome

e) Klinefelter syndrome

6) Gender identity disorder (GID) is a condition in which a person has been born one

gender, usually on the basis of their sex at birth, but identifies as belonging to another

gender, and feels significant discomfort or the inability to deal with this condition.

a) True

b) False

7 – Attraction and Love-Binding Forces

Not Covered

8 – Relationships and Communications

Not covered

Page 21: Quiz Human Sexuality

Human Sexuality 14Mar2009

DO NOT DISTRIBUTE - 21 -

9 – Sexual Techniques and Behavior Patterns

1.1) Early Judeo-Christians viewed masturbation as:

a) A sin

b) Something to be ignored

c) A natural act

d) A disgrace to the family

e) Symbolism of God

1.2) Many people, such as Slyvester Graham (Graham cracker), Dr. J. J. Kellogg

(cereal), Benjamin Rush, Krafft-Ebring, and others, believe that masturbation was

associated with disease processes.

a) True

b) False

2) Which of the following is NOT true regarding masturbation?

a) Men are more likely to masturbate than women

b) Married people are less likely to masturbate

c) People with more education masturbate more frequently

d) African Americans are the most likely ethnicity to masturbate

3.1) During sexual activity, women are more likely to fantasize about past sexual partners

than men.

a) True

b) False

3.2) One study found that ____ of men and ____ of women fantasize about someone

other than their sex partner.

a) 98%; 80%

b) 52%; 40%

c) 40: 52%

d) 80%; 98%

4) Which of the following is NOT a common foreplay technique?

a) Touching erogenous zones

b) Genital stimulation

c) Breast stimulation

d) Kissing

e) Vaginal insertion

5) Which ethnicity is the most likely to engage in oral-genital stimulation?

a) Asian Americans

b) South Americans

c) European Americans

d) African Americans

e) Australians

6) More highly educated individuals are more likely to have practiced oral sex.

a) True

b) False

7) Which of the following is the most frequent reason for abstaining from oral sex?

a) Odor

b) Taste

c) Smell

Page 22: Quiz Human Sexuality

Human Sexuality 14Mar2009

DO NOT DISTRIBUTE - 22 -

d) Texture

e) Shyness and embarrassment

8.1) Which of the following sexual positions is NOT recommended for pregnant women?

a) Male superior

b) Female superior

c) Lateral entry

d) Rear entry

8.2) Which position can be awkward for penile entry and may be difficult in achieving

deep penetration?

a) Male superior

b) Female superior

c) Lateral entry

d) Rear entry

8.3) Which position is most associated with queefing (vaginal flatulence) after sex?

a) Male superior

b) Female superior

c) Lateral entry

d) Rear entry

9) Coital fantasies allow couples to inject sexual variety without being unfaithful, and

there does not appear to be any connection between sexual dissatisfaction with one’s

relationship and coital fantasies.

a) True

b) False

10.1) More highly educated individuals are more likely to have practiced anal sex.

a) True

b) False

10.2) Which of the following should be used with anal sex to help prevent STDs?

a) Spermicidal lubricant

b) Water-based lubricant

c) Petroleum jelly

d) Condoms

e) The pull-out method

11.1) Married people rarely, if ever, masturbate.

a) True

b) False

11.2) European American men are more likely to masturbate than African American men.

a) True

b) False

11.3) Women who refrained from masturbation during adolescence are more likely to

find gratification in marital coitus.

a) True

b) False

11.4) Women achieve orgasm more reliably through coitus than masturbation.

a) True

b) False

11.5) Most women masturbate by inserting a finger or other object into the vagina.

Page 23: Quiz Human Sexuality

Human Sexuality 14Mar2009

DO NOT DISTRIBUTE - 23 -

a) True

b) False

11.6) Heterosexual people do not fantasize about sexual activity with people of their own

gender.

a) True

b) False

11.7) Statistically speaking, oral sex is the norm for today’s young married couples.

a) True

b) False

11.8) When lovers fantasize about other people, their relationship is in trouble.

a) True

b) False

11.9) Anal sex is more common among less educated people.

a) True

b) False

12.1) Coitus interruptus refers to:

a) Using condoms for sexual intercourse

b) Stopping coitus due to bodily noises

c) Pulling out during coitus prior to male orgasm

d) Premature ejaculation during coitus

12.2) Anilingus refers to:

a) Marriage without oral sex

b) A sexual encounter not involving oral sex

c) Anal sex with oral sex involving female genitalia

d) Anal-oral sex

10 – Sexual Orientation

1) One’s gender identity is where their sexual attraction and romantic interest lie.

a) True

b) False

2.1) On the Kinsey continuum of sexual orientation, what does a score or category of six

(6) designate?

a) Bisexual male

b) Bisexual female

c) Exclusively heterosexual

d) Exclusively bisexual

e) Exclusively homosexual

2.2) In the Storm two-dimensional model of sexual orientation, who would be described

as being high in both dimensions?

a) Asexuals

b) Heterosexuals

c) Homosexuals

d) Bisexuals

3.1) A female individual who was equally attracted to both men and women would be

called:

a) Bi-gay

Page 24: Quiz Human Sexuality

Human Sexuality 14Mar2009

DO NOT DISTRIBUTE - 24 -

b) Bi-straight

c) Bi-bi

d) Biphobic

e) Heterosexual

3.2) Bisexuals are more accepted socially than homosexuals.

a) True

b) False

4) Which of the following is NOT true regarding historical and religious views?

a) In Ancient Greece, men frequently formed relationships with adolescent males

b) The Romans described feminine gay men who dressed flamboyantly and

searched certain neighborhoods for partners

c) In 15th Century Florence, male-male sexual activity was a sin

d) In the Book of Leviticus, male-male sexual activity was recommended

e) In most of Western Europe, there were penalties for non-procreative sexual acts

5.1) Many preliterate societies viewed male-male sexual interactions as normal.

a) True

b) False

5.2) Over half of American adults believe that homosexuality is morally wrong.

a) True

b) False

5.3) Among Sambian people of New Guinea, 7-10 year old males leave their parent’s

house to undergo a sexual rites passage, which includes ingestion of semen, which is

believed to give rise to puberty.

a) True

b) False

6) People who have a strong stake in maintaining stereotypical gender roles (e.g. using

derogatory names, taunting, barring gay people from events) may feel more readily

threatened by the existence of the gay male or lesbian sexual orientation, because gay

people appear to confuse or reverse those roles.

a) True

b) False

7) The 1986 Supreme Court decision Hardwick vs. Bowers, which was struck down in

2003, let stand what Georgia sodomy law?

a) Male-male public relations, such as kissing, was punishable by up to 20 years

in prison

b) Male-male sexual contact was punishable by up to 20 years in prison

c) Oral-genital or anal-genital sexual contact was punishable by up to 20 years in

prison

d) Male-male oral-genital sexual contact was punishable by up to 20 years in

prison

e) Male-male anal-genital sexual contact was punishable by up to 20 years in

prison

8) Which of the following statements is true regarding sexual orientation?

a) In twin studies, identical twins (monozygotic) were not found to be concordant

for gay male sexual orientation when compared to non-twins

Page 25: Quiz Human Sexuality

Human Sexuality 14Mar2009

DO NOT DISTRIBUTE - 25 -

b) It has been proven that no X chromosomal region predisposes men to gay male

sexual orientation

c) The third interstitial nucleus of the anterior hypothalamus is smaller in gay

men than in heterosexual men

d) Genitals of gay people do not differentiate prenatally in accord with their

chromosomal sex as other factors modify this differentiation

e) Research has shown sexual orientation with levels of male or female sex

hormone in adulthood

9.1) Sigmund Freud felt gay male or lesbian sexual orientation resulted from failure to

resolve the Oedipus complex.

a) True

b) False

9.2) The majority of gay males and lesbians were not aware of sexual interest in people

of their own sex before they had sexual encounters with them.

a) True

b) False

10) Failure to form the gender-role stereotype that is consistent with one’s anatomic sex

is also known as:

a) Being a “tom boy”

b) Lesbianism

c) Gender confusion

d) Gender reversal

e) Gender nonconformity

11.1) Homosexuals have a lower likelihood of suicide than heterosexuals.

a) True

b) False

11.2) Therapy for homosexual men and women who wished to be heterosexual were:

a) Relatively successful

b) Neither successful nor unsuccessful

c) Relatively unsuccessful

d) Completely failures

12) Which of the following is considered the first step in the “coming out” process?

a) Disclosure of one’s sexual orientation to other people

b) Sexual contact with members of the same sex

c) Self-labeling as gay or lesbian

d) Attraction to members of the same sex

13.1) 79% of gay males reported having sex with strangers and 84% reported having at

least how many sexual partners in their lifetime?

a) 10

b) 20

c) 30

d) 40

e) 50

13.2) Lesbians are much less likely to have sex with strangers and have a significant

number of sexual partners.

a) True

Page 26: Quiz Human Sexuality

Human Sexuality 14Mar2009

DO NOT DISTRIBUTE - 26 -

b) False

14) Gay men are more likely to have committed, affectionate relationships than lesbians.

a) True

b) False

15.1) Gay males and lesbians would prefer to be members of the other sex.

a) True

b) False

15.2) Members of ethnic minority groups in the United States are less tolerant of

homosexuals than European Americans.

a) True

b) False

15.3) Today, the majority of Americans oppose gay marriages.

a) True

b) False

15.4) A majority of Americans believe that gay people choose to be gay.

a) True

b) False

15.5) The American Psychiatric Association considers homosexuality to be a mental

disorder.

a) True

b) False

15.6) Many gay couples have lifestyles similar to those of committed heterosexual

couples and are well adjusted.

a) True

b) False

16.1) Activating effects influence sex drive and sexual orientation.

a) True

b) False

16.2) The association male-female intercourse with fear of castration (castration anxiety)

has been scientifically demonstrated.

a) True

b) False

16.3) The term femme refers to:

a) A gay male who is masculine

b) A gay male who is feminine

c) A lesbian who is masculine

d) A lesbian who is feminine

e) A straight male who is very weak

11 – Conception, Pregnancy, and Childbirth

1) Which of the following methods of optimizing the chances of conception is the easiest

for couples to try?

a) Using a basal body temperature chart

b) Analyzing urine and saliva for leutenizing hormone (LH)

c) Tracking vaginal mucus

d) Modifying sexual position

Page 27: Quiz Human Sexuality

Human Sexuality 14Mar2009

DO NOT DISTRIBUTE - 27 -

2) Which of the following methods to select offspring gender is the most successful?

a) Shettler approach

b) Sperm separation

c) Body temperature

d) Modifying sexual position

e) Preimplantation genetic diagnosis

3.1) Low sperm count, which is normally 40-150 million sperm per milliliter, is the most

common reason for male infertility. Which of the following would cause low sperm

count?

a) Wearing loose underwear such as boxer briefs

b) Exercising 30 minutes a day, 3 days a week

c) Eating a well-balanced diet, including meat and dairy

d) Engaging in sexual activity multiple times a day, most days of the week

e) Spending 3 days a week relaxing in a hot tub

3.2) A female presents with infertility issues. It is found that one of her ovarian tubes is

blocked. Which of the following is the most likely cause?

a) Malformation of the Mullerian ducts

b) Diabetes

c) Endometriosis

d) Recurrent urinary tract infections (UTIs)

e) Increased hormone levels of estrogen and progesterone

4.1) A male has very low sperm count and wishes to have offspring with his wife. Which

of the following is the most reliable method for this couple?

a) Gamete intrafallopian transfer (GIFT)

b) Zygote intrafallopian transfer (ZIFT)

c) Donor in vitro fertilization

d) Embryonic transfer

e) Intracytoplasmic sperm injection (ICSI)

4.2) Which of the following methods involves combining sperm and ova in a laboratory

dish prior to implantation?

a) Gamete intrafallopian transfer (GIFT)

b) Zygote intrafallopian transfer (ZIFT)

c) Donor in vitro fertilization (Donor IVF)

d) Embryonic transfer

e) Intracytoplasmic sperm injection (ICSI)

5) Which of the following is true of sex during pregnancy?

a) Many women will show a decline in sexual interest during the first trimester

b) Sex can occur during the first trimester even if there is a risk of bleeding

c) The male-superior position is recommended during second trimester

d) There are no restrictions on sex during the third trimester

e) It is recommended to have sex if pregnant and menstruating

6) How long after pregnancy should a couple wait before continuing their sexual activity?

a) Two weeks

b) One month

c) Six weeks

d) Two months

Page 28: Quiz Human Sexuality

Human Sexuality 14Mar2009

DO NOT DISTRIBUTE - 28 -

e) Three months

7.1) Prolonged athletic activity may decrease fertility in males.

a) True

b) False

7.2) A “test tube baby” is grown in a laboratory disk throughout the nine months of

gestation.

a) True

b) False

7.3) There is an all-female species of lizard that lays unfertilized eggs that develop into

identical females generation after generation.

a) True

b) False

7.4) Morning sickness is a sign that pregnancy is progressing normally.

a) True

b) False

7.5) For the first week of conception, a fertilized egg cell is not attached to its mother’s

body.

a) True

b) False

7.6) Pregnant women can have an alcoholic beverage a day without harming their babies.

a) True

b) False

7.7) The way that the umbilical cord is cut determines whether the baby will have an

“inny” or an “outy.”

a) True

b) False

7.8) In the United states, nearly one in four births are done by cesarean section.

a) True

b) False

8) Which of the following would be best for a couple who wishes to have children but the

female does not produce ova?

a) Gamete intrafallopian transfer (GIFT)

b) Zygote intrafallopian transfer (ZIFT)

c) Donor in vitro fertilization (Donor IVF)

d) Embryonic transfer

e) Intracytoplasmic sperm injection (ICSI)

12 – Contraception and Abortion

1) Which of the following methods of contraception was NOT used in the past?

a) Dung soaked in sour milk

b) Animal intestine condoms

c) Wine and garlic douche

d) Coitus interruptus (withdrawal method)

e) Holy water on the belly and prayer

2.1) What is the role of progesterone in birth control pills?

a) Decreases FSH so follicles mature more quickly

Page 29: Quiz Human Sexuality

Human Sexuality 14Mar2009

DO NOT DISTRIBUTE - 29 -

b) Increases FSH so follicles do not mature

c) Decreases LH so ovulation is halted

d) Increases LH so ovulation occurs quickly

e) Causes muscles to contract and close the ovarian tubes

2.2) Which of the following is NOT true regarding birth control pills?

a) They have failure rates are below 3%

b) They increase the risk of blood clots

c) They reduce the effectiveness of certain antibiotics

d) They help prevent STDs

e) They can help reduce the risk of pelvic inflammatory disease

3) Which of the following contraception methods, which was discontinued in the United

States, involves inserting rods into the upper arm, which release progestin into the

bloodstream for five years?

a) RU486 (Mifepristone)

b) Norplant

c) Depo-provera (DMPA)

d) Nuva ring

e) IUD

4) Which of the following is the most serious risk of intrauterine devices (IUDs)?

a) Cramping

b) Uterine perforation

c) Pelvic inflammatory disease

d) Abnormal bleeding

e) Painful insertion

5) Which of the following has the highest contraceptive failure rate?

a) Birth control minipill

b) Birth control multiphasic pill

c) Norplant

d) IUD

e) Diaphragm

6) How long prior to intercourse should spermacides be applied?

a) 45-minutes

b) 30-minutes

c) 15-minutes

d) 10-minutes

e) 5-minutes

7) Why was the contraceptive sponge taken off the market in the United States in 1994?

a) High incidence of failure

b) Extreme difficulty in insertion

c) Incidence of getting stuck in the cervix

d) Bacterial contamination

e) Penile injury during sexual intercourse

8) The cervical cap contraceptive is like a diaphragm but must be fitted by a clinician.

a) True

b) False

9) When used alone, condoms (prophylactics) have about what failure rate?

Page 30: Quiz Human Sexuality

Human Sexuality 14Mar2009

DO NOT DISTRIBUTE - 30 -

a) 1%

b) 2%

c) 3%

d) 10%

e) 20%

10) Coitus interruptus (withdrawal method) has a 20% failure rate. Douching, which can

actually propel sperm into the uterus, has what failure rate for contraception?

a) 10%

b) 20%

c) 40%

d) 60%

e) 80%

11) Which of the following fertility awareness techniques has about a 20% failure rate in

the first year and involve testing urine for LH?

a) Calendar method (peak days)

b) Basal body temperature (BBT)

c) Cervical mucous (ovulation) method

d) Ovulation-Prediction kits

12) Years after a vasectomy procedure, a man remarries and wants to have children.

Which of the following complications of the vasovasectomy procedure can occur, leading

to difficulty conceiving?

a) The body can no longer produce sperm

b) The body can produce antibodies toward the sperm

c) The vasculature to the vas deferens has been damaged and thus they are no

longer viable

d) The vas deferens are permanently blocked

e) The sperm no longer have an affinity toward the ova

13) The contraceptive that involves a long-acting, synthetic form of progesterone is

known as:

a) RU486 (Mifepristone)

b) Norplant

c) Depo-provera (DMPA)

d) Nuva ring

e) IUD

14.1) Ancient Egyptians used crocodile dung as a contraceptive.

a) True

b) False

14.2) There is an oral contraceptive that can be taken two days after unprotected

intercourse.

a) True

b) False

14.3) Sterilization operations can be surgically reversed.

a) True

b) False

14.4) Testosterone can be used as a male contraceptive.

a) True

Page 31: Quiz Human Sexuality

Human Sexuality 14Mar2009

DO NOT DISTRIBUTE - 31 -

b) False

14.5) Abortions were legal in the newly founded United States.

a) True

b) False

14.6) Contraceptives prevent contraception as well as the spread of sexual transmitted

infections (STIs).

a) True

b) False

14.7) The dilation and curette (D & C) is the most widely used abortion method in the

United States.

a) True

b) False

15.1) Which of the following is true of the contraceptive minipill?

a) Contains synthetic progesterone and estrogen

b) Contains synthetic progesterone

c) Contains synthetic estrogen

15.2) Which of the following is a kind of tubal sterilization in which the fallopian tubes

are approached through an incision in the back wall of the vagina.

a) Complete hysterectomy

b) Partial hysterectomy

c) Culpotomy

d) Hysterotomy

e) Laproscopy

f) Minilaparotomy

15.3) The dilation and evacuation (D & E), is a common abortion method in which the

cervix is dilated prior to vacuum aspiration

a) True

b) False

16.1) The best time to talk with your partner about contraception is anytime that allows

your contraceptive to become effective before you engage in coitus.

a) True

b) False

16.2) The majority of Americans feel that abortion should not occur after the first two

trimesters.

a) True

b) False

16.3) Most Americans believe abortion should be permitted but there is great controversy

over partial-birth abortion.

a) True

b) False

13 – Sexuality in Childhood and Adolescence

1) Which of the following is the final indicator for infant sexual response?

a) Sexual curiosity

b) Genital play

c) Masturbation

Page 32: Quiz Human Sexuality

Human Sexuality 14Mar2009

DO NOT DISTRIBUTE - 32 -

2) When does masturbation typically begin for humans?

a) 3 to 6 months

b) 6 to 12 months

c) 2 years old

d) 4 years old

e) 6 years old

3) Scolding and punishing children who masturbate has no long-term affect on sexual

behavior.

a) True

b) False

4.1) According to Friedrich’s study, what is a common behavior for children ages 2 to 5?

a) Asking about kissing

b) Touching their mother’s breasts

c) Touching their private parts at home

d) Touching the private parts of relatives

e) Touching the breasts of strangers

4.2) According to Friedrich’s study, what is a common behavior for children ages 6 to 9?

a) Asking about kissing

b) Touching their mother’s breasts

c) Touching their private parts at home

d) Touching the private parts of relatives

e) Touching the breasts of strangers

5) Same sex play during childhood, such as touching genitals or oral-genital contact, does

not foreshadow adult sexual orientation.

a) True

b) False

6) The problem with most sex education modules in America is that it mainly focuses on

biological aspects of puberty and reproduction, but rarely deals with abortion,

masturbation, sexual orientation, or sexual pleasure.

a) True

b) False

7) Which of the following is NOT a suggestion by Calderone and Johnson for improving

parent-child communication about sexuality?

a) Be willing to answer questions about sex

b) Use necessary, and even vulgar, language

c) Give advice in the form of information that the child can use to make sound

decisions upon

d) Share information in small doses

e) Encourage the child to talk about sex

8) Why has the age of menarche declined among girls in Western nations?

a) An increase in fast-food diets

b) The use of alcohol at earlier ages

c) A decrease in physical activity

d) Teens are having sex more often

e) Better nutrition and health care

9) In what time frame does pubic hair begin to grow and cover the mons?

Page 33: Quiz Human Sexuality

Human Sexuality 14Mar2009

DO NOT DISTRIBUTE - 33 -

a) Ages 8 - 11

b) Ages 9 - 15

c) Ages 10 - 16

d) Ages 12 - 19

10) In what time frame does the first ejaculation occur?

a) Ages 9 - 15

b) Ages 11 - 16

c) Ages 11 - 17

d) Ages 14 - 18

11.1) African American males are more likely than Latino American and European

American males to have premarital sexual intercourse.

a) True

b) False

11.2) African American males are more likely than Latino American and European

American males to have engaged in cunnilingus.

a) True

b) False

12) Which of the following is a biological reason for adolescence engaging in sex?

a) To punish their parents

b) As a reward to guys for being loyal

c) Sexual arousal hormones

d) Because of love or desire

e) To show a dominant role

13) Which of the following would NOT decrease the likelihood that female adolescents

will be sexually active?

a) Adolescents that are more career orientated

b) Children in a two-parent household

c) Drug use

d) Family values

e) Religion

14) About what percent of adolescence report sexual experiences with people of their

own sex?

a) 1%

b) 2%

c) 5%

d) 10%

e) 50%

15) The recurrent themes for teen mothers are poverty, joblessness, and a lack of hope for

the future, with half of them quitting school. What percent of American girls between the

ages of 15 and 19 become pregnant each year?

a) 1%

b) 2%

c) 5%

d) 10%

e) 50%

Page 34: Quiz Human Sexuality

Human Sexuality 14Mar2009

DO NOT DISTRIBUTE - 34 -

16) Which of the following is NOT a factor that contributes to the incidence of teen

pregnancy?

a) Loosening of traditional taboos

b) Lack of religion

c) Impaired family relationships

d) Problems in school

17) Adolescents often report not using contraceptives because they feel that not having

intercourse often enough warrants their use. They also report not wanting to disrupt sex

with the use of contraceptives.

a) True

b) False

18) Most adolescents admit that it would be easier for them to postpone sex and avoid

pregnancy if they could have more open discussions with their parents.

a) True

b) False

19.1) Many boys are born with erections.

a) True

b) False

19.2) Most children learn the facts of life from their parents or from school sex education

programs.

a) True

b) False

19.3) There is evidence that sex education encourages sexual activity.

a) True

b) False

19.4) Erotic dreams in adolescent men always result in nocturnal emission (wet dreams).

a) True

b) False

19.5) Sexual petting is universal among adolescents in the United States.

a) True

b) False

19.6) Nearly 800,000 adolescent girls in the United States become pregnant each year.

a) True

b) False

20.1) The Critical Fat Hypothesis is a view that girls must reach a certain body weigh to:

a) Being sexual contact with the opposite sex

b) Develop secondary sex characteristics

c) Reach menarche

d) Feel comfortable with sexual contact

e) Be sexually attractive to boys

20.2) Personal fable, the belief that one’s feelings and ideas are special and that one is

invulnerable, is most common amongst adolescence.

a) True

b) False

21.1) Communication with your children about tender subjects such as sex is nearly

always a good idea.

Page 35: Quiz Human Sexuality

Human Sexuality 14Mar2009

DO NOT DISTRIBUTE - 35 -

a) True

b) False

21.2) Most girls are satisfied with their first sexual encounter.

a) True

b) False

22) Because of the lack of education, young African Americans are paying a very steep

price with an increase in STDs including HIV/AIDS.

a) True

b) False

23) Internet access and technology is a double-edged sword for parents today. It allows

parents to learn about adolescent sexual topics but also provides a means for teenagers to

access adult content.

a) True

b) False

14 – Sexuality in Adulthood

1) Which of the following types of marriage is legal in the United States?

a) Monogamy

b) Polygyny

c) Polyandry

d) B & C

e) A, B, & C

2) Which of the following in 2008 does NOT allow full marriage rights to same sex

couples?

a) Netherlands

b) Belgium

c) Canada

d) United States

3) Today, married couples engage in coitus more frequently, with greater variety, and for

longer periods of time.

a) True

b) False

4) What reason do women give the most for participating in extramarital affairs?

a) More frequent sex

b) Better sex

c) Friendship

d) Love

e) Romance

5) As the composition of the U.S. population gets progressively older, it is important to

be aware of sex in the elderly, as it is important to preventing problems that may arise.

a) True

b) False

6) With fine tuning of expectations, elderly couples may find themselves leading some of

the most sexually fulfilling years of their lives.

a) True

b) False

Page 36: Quiz Human Sexuality

Human Sexuality 14Mar2009

DO NOT DISTRIBUTE - 36 -

7) Which of the following does NOT often occur with aging?

a) Reduced vaginal lubrication

b) Reduced female orgasm intensity

c) More semen production due to prostate

d) Softer erections

e) Longer refractory periods

8) The availability of a sexually interested and supportive partner may be the most

important determinant of continued sexual activity.

a) True

b) False

9) Which of the following is NOT a factor that Nosek identifies as central to sexual

wellness among the disabled?

a) Positive sexual self-concept

b) Knowledge about sexuality

c) Positive, productive relationships

d) Ignoring current barriers to sexuality

e) Maintaining the best possible general and sexual health

10) Which of the following disabilities leads to inability to perform sexually?

a) Cerebral palsy

b) Lose vaginal sensation or lubrication

c) Spinal cord injury males

d) Arthritis

e) None of the above

11) Regardless of physical, sensory, or psychological disability, people just need time to

develop a comfort zone that applies to their sexuality.

a) True

b) False

12.1) Singlehood has become a more common U.S. lifestyle over the past few decades.

a) True

b) False

12.2) Divorced people are more likely than never married people to cohabitate.

a) True

b) False

12.3) In the ancient Hebrew and Greek civilizations, wives were viewed as their

husband’s property.

a) True

b) False

12.4) Men are more romantic, with regard to love at first sight and believing in soul

mates, than women.

a) True

b) False

12.5) Most of today’s sophisticated young people see nothing wrong with an occasional

extramarital fling.

a) True

b) False

12.6) Men are more likely than women to commit acts of domestic violence.

Page 37: Quiz Human Sexuality

Human Sexuality 14Mar2009

DO NOT DISTRIBUTE - 37 -

a) True

b) False

12.7) Few women can reach orgasm after the age of 70.

a) True

b) False

12.8) People who are paralyzed as a result of spinal cord injuries cannot become sexually

aroused or engage in coitus.

a) True

b) False

13.1) Swinging or “mate-swapping” is also known as:

a) Polygamy

b) Polyandry

c) Polygyny

d) Comarital sex

e) Extramarital sex

13.2) The mating gradient is the tendency for women to “marry-up” and for men to

“marry-down” with regard to social and economic status.

a) True

b) False

15 – Sexual Dysfunctions

1) Which of the following Diagnostic and Statistical Manual of Mental Disorders (DMD)

categories of sexual dysfunction involves dyspareuia?

a) Sexual desire disorders

b) Sexual arousal disorders

c) Orgasm disorders

d) Sexual pain disorders

2.1) Hypogonadism (small testes) or removal or the adrenal gland could result in which

type of disorder?

a) Sexual desire disorder

b) Sexual arousal disorder

c) Orgasm disorder

d) Sexual pain disorder

2.2) If a women can reach orgasm during masturbation or oral sex, they can reach orgasm

through coitus with their partner.

a) True

b) False

2.3) Vaginismus, which involves involuntary contraction of the pelvic muscles that

surround the outer third of the vaginal barrel and is most often caused by a psychological

fear of penetration, is considered which type of disorder?

a) Sexual desire disorder

b) Sexual arousal disorder

c) Orgasm disorder

d) Sexual pain disorder

3) Which of the following organic factors is most associated with erectile dysfunction?

a) Alcohol use

Page 38: Quiz Human Sexuality

Human Sexuality 14Mar2009

DO NOT DISTRIBUTE - 38 -

b) Heroine use

c) High cholesterol

d) Morphine

e) Cocaine use

4) Which of the following factors would NOT cause sexual dysfunction?

a) Emotional factors

b) Psychosexual trauma

c) Sexual orientation

d) Lack of sexual skill

e) Familial factors

5) What is the goal of sex therapy?

a) To help the couple achieve orgasm

b) To help the couple procreate

c) To introduce the couple to new sexual experiences

d) To incorporate sexual toys

e) To change self-defeating beliefs and attitudes

6) When looking for a sex therapist, it is important to find someone with specialized

training and current certification and licensure.

a) True

b) False

7.1) Alprostadil is used to treat sexual arousal disorder via what application?

a) Applying to the anus

b) Inserting into the penile urethra

c) Ingestion (pill)

d) Injection (IM)

e) Transdermal patch

7.2) How does Apomorphine (Uprima) treat sexual arousal disorders?

a) Increases dopamine, leading to erection

b) Decreases dopamine, leading to erection

c) Increase norepinephrine, leading to vaginal secretions

d) Decreases norepinephrine, leading to vaginal secretions

8) A testosterone patch can enhance sexual arousal in females.

a) True

b) False

9.1) A common method of treating sexual orgasm disorders is a couples-orientated

approach where the woman guides the man in caressing and movements.

a) True

b) False

9.2) What is the role of clomipramine in sexual disorders?

a) Increases the strength of female orgasm

b) Increases the strength of male orgasm

c) Prevents premature female orgasm

d) Prevents premature male orgasm

e) Allows for deeper coital penetration

10) Dyspareuina (painful intercourse) is usually treated by managing the underlying

physical problem, such as a urinary tract infection.

Page 39: Quiz Human Sexuality

Human Sexuality 14Mar2009

DO NOT DISTRIBUTE - 39 -

a) True

b) False

11) The development of treatments for sexual dysfunction for women has lagged behind

men due to a lack of understanding of the female sexual dysfunctions and it has been

historically a male driven society for drugs of this nature to get attention.

a) True

b) False

12.1) Sexual dysfunctions are rare.

a) True

b) False

12.2) Only men can reach orgasm too early.

a) True

b) False

12.3) The most common cause of painful intercourse in women is vaginal infection.

a) True

b) False

12.4) Sex therapy teaches a man with erectile disorder how to will an erection.

a) True

b) False

12.5) A doctor made a somewhat unusual presentation to a medical convention by

dropping his pants to reveal an erection.

a) True

b) False

12.6) Many sex therapists recommend masturbation as the treatment for women who

have never been able to reach orgasm.

a) True

b) False

12.7) A man can be prevented from ejaculating by squeezing his penis when he feels the

he is about to ejaculate.

a) True

b) False

13.1) A spectator role, in which people observe rather than fully participate in their

sexual encounters, is usually taken because of performance anxiety.

a) True

b) False

13.2) Exercises in which sex partners take turns giving and receiving pleasurable

stimulation in nongenital areas of the body are called:

a) Tantric

b) Tumescent

c) Squeeze

d) Sesate focus

e) Vaginismus

14.1) Dhat Syndrome is found among young Asian males and involves excessive fears

over the loss of seminal fluid during nocturnal emissions.

a) True

b) False

Page 40: Quiz Human Sexuality

Human Sexuality 14Mar2009

DO NOT DISTRIBUTE - 40 -

14.2) Comparing cultures, in Mangia premarital relations do not occur and the Inis Beag

are taught sexual techniques are a young age.

a) True

b) False

16 – Sexually Transmitted Infections

1) The United States is believed to have the highest rate of sexually transmitted disease

(STIs) in the industrialized world

a) True

b) False

2.1) Which of the following is considered a fungal infection?

a) Vaginosis (BV)

b) Chlamydia

c) Tricomoniasis

d) Syphilis

e) Gonorrhea

2.2) Which of the following is considered a fungal infection?

a) Genital herpes

b) Hepatitis

c) HIV

d) Genital warts

e) Candidiasis

2.3) Viral herpes can be cured with drugs such as acyclovir and famciclovir.

a) True

b) False

3) Which of the following is transmitted exclusively with sexual contact?

a) Scabies

b) Pediculosis (crabs)

c) Genital warts

d) Candidiasis

e) Syphilis

Match the common symptom with the disease:

4.1) Yellow, thick, burning penile discharge a) Scabies

4.2) Hard, round painless chancre b) Crabs

4.3) Painful urination, mostly asymptomatic c) Warts

4.4) Thin, foul smelling discharge d) HIV

4.5) Vulva itching, foul discharge e) Hepatitis

4.6) Foamy, yellow, odorous discharge f) Herpes

4.7) Painful, reddish bumps, prodromal symptoms g) Tricomoniasis

4.8) Asymptomatic to mild flu-like symptoms h) Candidiasis

4.9) Fever, weight loss, rare forms of cancer i) Bacterial vaginosis

4.10) Painless warts, cauliflower shaped j) Chlamydia

4.11) Itching in pubic hair area k) Syphilis

4.12) Intense itching, reddish lines on skin l) Gonorrhea

5.1) Which of the following requires 3 of 4 diagnostic markers and includes Clue cells?

a) Gonorrhea

Page 41: Quiz Human Sexuality

Human Sexuality 14Mar2009

DO NOT DISTRIBUTE - 41 -

b) Bacterial vaginosis

c) Tricomoniasis

d) HIV

e) Crabs

5.2) How is genital herpes diagnosed?

a) Fluid drawn from the base of a genital sore

b) Microscopic examination

c) Abbott Testpack

d) VDRL blood test

e) Antibodies

5.3) How is viral hepatitis diagnosed?

a) Fluid drawn from the base of a genital sore

b) Microscopic examination

c) Abbott Testpack

d) VDRL blood test

e) Antibodies

Match the disease with the drug of choice:

6.1) Gonorrhea a) Acyclovir

6.2) Syphilis b) Metronidazole

6.3) Chlamydia c) Ceftriaxone

6.4) Bacterial vaginosis d) Penicillin

6.5) Herpes e) Azithromycin

7) Which of the following could most help reduce the risk of spreading STDs?

a) Anal rather than vaginal intercourse

b) Using the withdrawal method

c) Birth control pills

d) Oral-genital stimulation

e) Condoms

8) Gardnerella vaginalis is associated with which of the following?

a) Gonorrhea

b) Tricomoniasis

c) Candidiasis

d) Bacterial vaginosis

e) Syphilis

9) What is the recommended treatment for genital warts?

a) Azithromycin

b) Ceftriaxone

c) Ciprofloxacin

d) Metronidazole

e) Acyclovir

f) Cryotherapy

g) Fluconazole

10) What is a likely result of a gonorrhea infection that is not treated?

a) Excessive ovulation

b) Pancreatitis

c) Pelvic inflammatory disease (PID)

Page 42: Quiz Human Sexuality

Human Sexuality 14Mar2009

DO NOT DISTRIBUTE - 42 -

d) Damage to renal nephrons

e) Blindness

11) A patient presents with a rash on their palms and foot soles. Blood tests are positive

for Syphilis. What stage is this patient in?

a) First

b) Second

c) Third

d) Early forth

e) Late forth

12) Which of the following has the highest risk for transmitting HIV?

a) Oral sex on a woman

b) Oral sex on a man

c) Needle drug use

d) Vaginal intercourse

e) Lesbian sexual interactions

13) HIV uses gp120 spikes to bind to sites on cells in the immune system, eventually

crippling the immune system. At what point does HIV become AIDS?

a) CD4 < 50 cells per cubic mm

b) CD4 < 100 cells per cubic mm

c) CD4 < 200 cells per cubic mm

d) CD4 < 500 cells per cubic mm

e) CD4 < 1,000 cells per cubic mm

14) HIV is often transmitted through insect and animal bites and can be transmitted in

work environment such as from bumping into someone or at drinking fountains.

a) True

b) False

15) Which of the following psychosocial factors would not underlie risky sexual behavior

among young people?

a) Perceived low risk of infection

b) Absense of religion in the household

c) Negative attitudes toward condom use

d) Feeling of personal invulnerability

16.1) Most women who contract gonorrhea do not develop symptoms.

a) True

b) False

16.2) Christopher Columbus brought more than beads, blankets, and tobacco back to

Europe from the New World: he also brought syphilis.

a) True

b) False

16.3) Gonorrhea and syphilis can be contracted from toilet seats in public rest rooms.

a) True

b) False

16.4) If a syphilitic sore goes away by itself, the infection does not require medical

treatment.

a) True

b) False

Page 43: Quiz Human Sexuality

Human Sexuality 14Mar2009

DO NOT DISTRIBUTE - 43 -

16.5) Men can develop vaginal infections.

a) True

b) False

16.6) Most people who are infected with HIV remain asymptomatic and appear healthy

for years.

a) True

b) False

16.7) Genital herpes can be transmitted only during flare-ups of infection.

a) True

b) False

17.1) HAART therapy is associated with which of the following?

a) Genital herpes

b) Hepatitis

c) HIV

d) Genital warts

e) Candidiasis

17.2) Donovanosis (granuloma inguinale) is a bacterial disease common in many

underdeveloped regions and has painless genital ulcers which can be mistaken for:

a) Candidiasis

b) Chlamydia

c) Tricomoniasis

d) Syphilis

e) Gonorrhea

17.3) What type of infection is Molluscum contagiosum?

a) Bacterial

b) Viral

c) Ectoparasite

d) Fungi

17.4) Which of the following causes Lymphogranuloma venereum (LGV)?

a) Candidiasis

b) Chlamydia

c) Tricomoniasis

d) Syphilis

e) Gonorrhea

17 – Atypical Sexual Variations

Covered in Behavioral Health Module

18 – Sexual Coercion

1) Although rare, when a women is involved in rape of another person she is often aiding

or abetting men who are attacking a woman.

a) True

b) False

2) Most men who rape other men are heterosexual and sexual motives are often absent.

a) True

b) False

Page 44: Quiz Human Sexuality

Human Sexuality 14Mar2009

DO NOT DISTRIBUTE - 44 -

3) Which of the following is NOT true of incarcerated rapists?

a) Most rapists are in control of their behavior

b) Most rapists know their behavior is illegal

c) Most rapists are mentally ill

d) Some rapists have long histories of violence

e) For some rapists, violence and sex become intermeshed

4) Which of the following is NOT a suggestion from Powell to provide for individuals

who have been raped?

a) Shower to help feel less humiliated

b) Consider reporting the incident to police

c) Ask a friend or relative to take you to the hospital

d) Seek help in an assertive way

e) Question health professionals about risks and treatments available

5) Child molesters and pedophiles are nearly always male.

a) True

b) False

6) Incest taboo helps reduce genetic defects and thus promotes survival of the species.

a) True

b) False

7) What is the most common form of incest?

a) Mother-son

b) Father-daughter

c) Brother-sister

8) A common scenario for incest is a dominant father who has been rejected sexually by

his wife, and turns to his daughter for emotional and sexual support.

a) True

b) False

9) Which of the following is NOT part of sexual harassment?

a) Deliberate unsolicited comments

b) Repeated unwelcome gestures

c) Unsolicited physical contact

d) Unwelcome contact

e) Lunch with a coworker of the opposite sex

10.1) A woman is raped every 10 minutes.

a) True

b) False

10.2) Rape is ten times greater in the U.S. than Japan.

a) True

b) False

10.3) A majority of rapes are committed by strangers.

a) True

b) False

10.4) When women say no, they sometimes mean yes.

a) True

b) False

10.5) Most rapists are mentally ill.

Page 45: Quiz Human Sexuality

Human Sexuality 14Mar2009

DO NOT DISTRIBUTE - 45 -

a) True

b) False

10.6) It is unclear whether women should attempt to fight off a rapist.

a) True

b) False

11) A rape that is motivated by the desire to control and dominate the person assaulted is

known as:

a) Acquaintance rape

b) Sadistic rape

c) Anger rape

d) Power rape

e) Statutory rape

19 – Commercial Sex

1) Those working for escort services as well as call girls are usually:

a) Well educated and from lower class families

b) Well educated and from middle class families

c) Well educated and from upper class families

d) Poorly educated and from lower class families

e) Poorly educated and from middle class families

2) Which of the following is NOT a reason why men seek to purchase services from

prostitutes?

a) Sex without negotiation or commitment

b) Sex for eroticism and variety

c) A way to socialize

d) Sex away from home

e) A way to prevent pregnancy

3.1) Prostitution is illegal throughout the entire United States.

a) True

b) False

3.2) Nearly all massage and escort services that advertise in the Yellow Pages are fronts

for prostitution.

a) True

b) False

3.3) It appears that the majority of female prostitutes were sexually abused as children.

a) True

b) False

3.4) Only males are sexually aroused by pornography and other prurient material.

a) True

b) False

3.5) Porn does not make men violent, but can make violent men more violent.

a) True

b) False

3.6) About 25% of all Internet surfing is devoted to the pursuit of porn, adult chat rooms,

and the like.

a) True

Page 46: Quiz Human Sexuality

Human Sexuality 14Mar2009

DO NOT DISTRIBUTE - 46 -

b) False

4.1) Men who engage in prostitution with male customers are called:

a) Call girls

b) Hustlers

c) Pimps

d) Scores

e) Streetwalkers

4.2) The Whore-Madonna complex is a rigid stereotyping of women as either sinners or

saints.

a) True

b) False

20 – Introduction To Human Sexuality & Adjunctive Material

1) Management of which of the following medical conditions can directly affect sexual

function?

a) Vascular disease

b) Neurological dysfunction

c) Diabetes

d) Arthritis

e) All of the above

2) In the Tanner Stages of development (Sex Maturity Ratings, SMRs), what stage is

denoted by pubic hair spreading to the medial surfaces of thighs?

a) SMR1

b) SMR2

c) SMR3

d) SMR4

e) SMR5

3) In general, SMR classification for boys uses their ____ and for girls uses their ____.

a) Penis; Vagina

b) Penis; Breasts

c) Testicles; Vagina

d) Testicles; Breasts

e) Voice; Self-image

4) Physicians should not assume their patients understand medical terms and thus should

use which of the following?

a) Simple language only

b) Simple language mostly and explicit language as necessary

c) Simple language and explicitly language freely

d) Simply language as necessary and explicit language mostly

e) Explicit language only

5) Which of the following should NOT be true when taking a sexual patient history?

a) Choose a comfortable setting

b) Care must be taken when choosing words to use

c) Assure the patient of confidentiality

d) Be judgmental

e) Have a respectful attitude

Page 47: Quiz Human Sexuality

Human Sexuality 14Mar2009

DO NOT DISTRIBUTE - 47 -

6) According to “The Proactive Sexual Health History,” what is the first screening

question to ask during a sexual history?

a) Have you ever been sexually involved with anyone?

b) What sexual concerns do you have?

c) Have you been sexually involved with anyone in the past six months?

d) With men, women, or both?

e) If you were sexually active, do you imagine it would be with men, women, or

both?

7) The estimated prevalence of sexual dysfunction (2002) in the general population is as

high as 52 percent in men and 63 percent in women. Sexual concerns have been reported

in 75% of couples seeking marital therapy and about what percentage of women seeking

routine gynecologic care?

a) 0%

b) 25%

c) 50%

d) 75%

e) 100%

8) Which of the following is NOT a preventive sexual health question?

a) Do you use needles to inject street drugs?

b) How do you protect yourself from HIV and other STDs?

c) Have you ever been tested for HIV? Would you like to be?

d) Do you use anything to prevent pregnancy? Are you satisfied with that

method?

e) Have you ever been immunized against hepatitis>? Would you like to be?

9) In the 2003 U.S. Supreme Court decision Lawrence vs. Texas, what type of sodomy

laws were invalidated in the fifty states?

a) Sex with dead bodies

b) Oral sex

c) Anal sex

d) Beastiality

e) Homosexual sex

10) The Center for Disease Control (CDC) shows that over what percentage of men and

women engage in heterosexual anal stimulation?

a) 5%

b) 10%

c) 15%

d) 20%

e) 30%

11) The Food and Drug Administration (FDA) approved Aneros device is meant to

stimulate which of the following?

a) Female G-spot (Grafenberg spot)

b) Penis

c) Testicles

d) Prostate

e) Clitoris

12) What are intra vaginal balls (Ben-Wa balls) used for?

Page 48: Quiz Human Sexuality

Human Sexuality 14Mar2009

DO NOT DISTRIBUTE - 48 -

a) Sexual stimulation for the female

b) Sexual stimulation for the male partner

c) Strengthening the pubococcygeus muscle

d) Preventing yeast infections

e) A & C

13) Penis rings are placed behind the testicles and over the dorsal shaft to prolong

erection. Damage can occur if these rings are used for longer than:

a) Thirty-seconds

b) One-minute

c) Five-minutes

d) Thirty-minutes

e) Twenty-four hours

14) What sexual position is recommended for pregnant or obese patients?

a) Rear-entry (“doggy style”)

b) Female superior (“cowgirl”)

c) Male superior (“missionary”)

e) Sitting (“lotus”)

15) What sexual position is recommended for a male patient who recently had a heart

attack?

a) Rear-entry (“doggy style”)

b) Female superior (“cowgirl”)

c) Male superior (“missionary”)

d) Lateral recumbent

e) Sitting (“lotus”)

16) Which of the following positions is the worst for women with back complaints?

a) Rear-entry (“doggy style”)

b) Female superior (“cowgirl”)

c) Male superior (“missionary”)

d) Lateral recumbent

e) Sitting (“lotus”)

17) Pre-menopausal women should perform their breast self-exam (BSE):

a) 5-10 days prior to menstruation

b) Just prior to menstruation

c) During menstruation

d) Just after menstruation

e) 5-10 days after menstruation

18) Which of the following symptoms in the “ACHES” pain mnemonic can signify

possible gallbladder disease

a) Abdominal pain

b) Chest pain

c) Headache

d) Eye problems

e) Severe leg pain

19) The CDC advices NOT to use which of the following associated with condoms?

a) Ribbed condoms

b) Water-based lubricant

Page 49: Quiz Human Sexuality

Human Sexuality 14Mar2009

DO NOT DISTRIBUTE - 49 -

c) Nonoxynol-9

d) Latex

e) Rubber

20) Increased vaginal lubrication as well as an engorged vagina are most likely seeing

during which trimester of pregnancy?

a) First

b) Second

c) Third

21) The CDC estimates that ____ of HIV infected patients are over the age of 50?

a) 0.5%

b) 1%

c) 2%

d) 5%

e) 10%

22) A 1979 study showed ____ lesbians came to terms with their sexuality after age 50.

a) All

b) 1 in 2

c) 1 in 3

d) 1 in 4

e) No

23) Which of the following is NOT a normal age-related sexual change?

a) Longer interval to ejaculate

b) Longer post orgasm refractory period for men

c) Delayed and more firm erection

d) Reduced vaginal expansion

e) Reduced vaginal lubrication

24) If a woman has bacterial vaginosis (BV) ____ time(s) with the same sex partner, than

the partner should be treated as well.

a) 1

b) 2

c) 3

d) 4

e) 5

25) A patient present with vaginal itching and burning. An elevated pH (>4.5) is found on

the anterior vaginal fornix, a positive KOH “whiff” test elicits a fish odor, and clue cells

are found under microscopy. Which of the following drugs should be given to this

patient?

a) Azithromycin

b) Ceftriaxone

c) Ciprofloxacin

d) Metronidazole

e) Acyclovir

f) Cryotherapy

g) Fluconazole

Page 50: Quiz Human Sexuality

Human Sexuality 14Mar2009

DO NOT DISTRIBUTE - 50 -

26) A patient presents with painful “kissing” ulcerations on their penis. Laboratory

results come back positive for Haemophilus ducreyi. What drug should the patient be

given for their chancroid?

a) Azithromycin

b) Ceftriaxone

c) Ciprofloxacin

d) Metronidazole

e) A or B

f) B or C

g) C or D

27) A patient presents with clear discharge and no other sexual symptoms. A DNA Probe

tests positive for Chlamydia trachomatis. What drug should this patient be given?

a) Azithromycin

b) Ceftriaxone

c) Ciprofloxacin

d) Metronidazole

e) Acyclovir

f) Cryotherapy

g) Fluconazole

28) If a patient is taking Metronidazole, what should they avoid doing?

a) Exercising vigorously

b) Eating fatty foods

c) Eating shellfish

d) Drinking alcohol

e) Smoking cigarettes

29) A patient presents with dysuria and penile discharge. Chocolate agar in the lab

reveals Neisseria gonorrhoeae. What drug should the patient be given?

a) Azithromycin

b) Ceftriaxone

c) Ciprofloxacin

d) Metronidazole

e) Acyclovir

f) Cryotherapy

g) Fluconazole

30) Patients who present with gonorrhea should also be treated for:

a) Herpes

b) HIV

c) Chlamydia

d) Bacterial vaginosis

e) Syphilis

31) A patient presents with painful genital blisters that come and go with time. What

should they be given?

a) Azithromycin

b) Ceftriaxone

c) Ciprofloxacin

d) Metronidazole

Page 51: Quiz Human Sexuality

Human Sexuality 14Mar2009

DO NOT DISTRIBUTE - 51 -

e) Acyclovir

f) Cryotherapy

g) Fluconazole

32) A patient presents with cauliflower-like genital warts and is diagnosed with human

papillomaviris (HPV). What is the recommended treatment?

a) Azithromycin

b) Ceftriaxone

c) Ciprofloxacin

d) Metronidazole

e) Acyclovir

f) Cryotherapy

g) Fluconazole

33) The drug pedophyllin should NOT be used with:

a) Penile warts

b) Scrotal warts

c) Labial warts

d) Perineal warts

e) Anal warts

34) Non-specific cervicitis (NSC) and mucopurulent cervicitis (MPC) require at least

____ white blood cells (WBCs) per high-power film (hpf) for diagnosing in females.

a) 2

b) 3

c) 5

d) 8

e) 10

35) Non-gonococcal urethritis (NGU) and non-specific urethritis (NSU) require at least

____ WBCs per hpf Gram stain for diagnosis in males.

a) 2

b) 3

c) 5

d) 8

e) 10

36) What drug is given for NSC and MPC?

a) Azithromycin

b) Ceftriaxone

c) Ciprofloxacin

d) Metronidazole

e) Penicillin

f) Fluconazole

37) Which of the following requires a broad range of suspicion for diagnosis and is the

leading cause of infertility and ectopic pregnancy?

a) Non-specific cervicitis (NSC)

b) Mucopurulent cervicitis (MPC)

c) Pelvic inflammatory disease (PID)

d) Trichomoniasis

e) Candidiasis

Page 52: Quiz Human Sexuality

Human Sexuality 14Mar2009

DO NOT DISTRIBUTE - 52 -

38) A patient presents with a rash on their palms and soles of their feet. History reveals

they had a genital chancre, but did not seek help because it was painless. What drug

should this patient be given as a single dose?

a) Azithromycin

b) Ceftriaxone

c) Ciprofloxacin

d) Metronidazole

e) Penicillin

f) Fluconazole

39) For late latent (greater than one year) or tertiary (non-neuro) Syphalis, how many

dosages of the recommended drug are given over consecutive weeks?

a) 1

b) 2

c) 3

d) 4

e) 5

40) A patient presents with green, sometimes gray, frothy discharge. Wet mount

visualization shows Trichomonas vaginalis. What drug should they be given?

a) Azithromycin

b) Ceftriaxone

c) Ciprofloxacin

d) Metronidazole

e) Penicillin

f) Fluconazole

41) Which of the following presents as a glans penis “rash,” may be associated with

uncircumcised males, and requires Mycolog (Nystatin and Triamcinolone Acetonide

Cream) treatment?

a) PID

b) Trichomoniasis

c) Balanitis

d) Candidiasis

e) Hepatitis

42) A female patient presents with a white creamy “cottage cheese” discharge. A KOH

prep is used to diagnose candidiasis (yeast infection). The patient says they tried a 3-day

vaginal cream last week. Which of the following drugs should be given as a single dose?

a) Azithromycin

b) Ceftriaxone

c) Ciprofloxacin

d) Metronidazole

e) Penicillin

f) Fluconazole

43) Which of the following is considered a low risk factor for Hepatitis C infection?

a) Multiple sexual partners

b) Injecting drug users

c) Hemodialysis patients

d) Recipients of clotting factors prior to 1987

Page 53: Quiz Human Sexuality

Human Sexuality 14Mar2009

DO NOT DISTRIBUTE - 53 -

e) Recipients of blood transfusions prior to 1992

44) Which of the following types of drugs are NOT associated with female sexual

dysfunction (FSD)?

a) Antipsychotics (e.g. SSRIs)

b) Antihypertensives

c) Anticholinergics

d) Hormonal preparations (e.g. Danazol)

e) IM testosterone

45) Which of the following types of drugs is associated with erectile dysfunction (ED)?

a) Cardiac

b) Antihypertensives

c) Antidepressants

d) Sedatives

e) Antiandrogens

f) All of the above

46) Which of the following drugs would be contraindicated with drugs such as Sildenafil

(Viagra), Vardenasil (Levitra), and Tadalafil (Cialis)?

a) Beta blockers

b) Digoxin

c) Lithium

d) Nitroglycerine

e) Dilantin

47) Which of the following is NOT true of body piercing recommendations by the

American Academy of Dermatology?

a) Universal precautions per the Occupational Safety and Health Administration

(OSHA) must be adhered to

b) Parental consent and supervision is required under the age of 18

c) Those getting pieced cannot have a blood alcohol content (BAC) > 0.10

d) Wound care instructions must be given and explained

e) Pre-piercing consultation must describe risks and complications

48) Sex in hot tubs is associated with which of the following?

a) Bacterial vaginosis

b) Pelvic inflammatory disease

c) Trichomoniasis

d) Balanitis

e) Candidiasis

49) Male masturbation with a vacuum cleaner has been shown to:

a) Be a safe form of masturbation

b) Cause local trauma but no lasting effects

c) Cause the penis to lengthen and increase in girth over time

d) Cause erectile dysfunction

e) Cause severe injury including decapitation and loss of the glans penis

50) Gerbilling, also known as gerbil stuffing or gerbil shooting, refers to the insertion of

small animals into a man's rectum to stimulate the prostate as in anal sex. Contrary to

popular belief, gerbilling is unknown as an actual sexual practice, and despite apparently

Page 54: Quiz Human Sexuality

Human Sexuality 14Mar2009

DO NOT DISTRIBUTE - 54 -

widespread public belief and persistent rumors, especially in the 1980s, no verified

medical evidence of gerbilling exists.

a) True

b) False

51) Hypoxic orgasm, which refers or masturbation activity with anoxia for a heightened

climax, as well as autoerotic asphyxia have been associated with unintentional brain

damage and death.

a) True

b) False

52) Bicycle riding has been associated with erectile dysfunction (ED).

a) True

b) False

53) Amyl nitrite (snappers, poppers) is inhaled by some to intensify sexual activities and

orgasm. The drug works as a potent:

a) Bronchodilator

b) Bronchocontrictor

c) Vasodilator

d) Vasoconstrictor

e) Muscle relaxant

54) What does “on the down low” mean regarding male prisoners?

a) They have performed oral sex on another prisoner

b) They have performed oral sex on a prison guard

c) They have had anal or oral sex and admit they are homosexual

d) They have had anal or oral sex and deny they are homosexual

e) They completely abstained from sexual activity while in prison

55) Men who are “on the down low” may not relate to preventative care messages created

for men who identify themselves as homosexual.

a) True

b) False

56) Which of the following sexual side effects is associated with men who take

Metoclopramide or Cimetidine?

a) Testicular swelling

b) Penile shrinking

c) Breast enlargement

d) Increased ejaculation

e) Difficulty achieving orgasm

57) Which of the following is the most likely sexual dysfunction in women with diabetes

mellitus (DM)?

a) Impaired vaginal lubrication

b) Vulval tumescence (swollen)

c) Increased sexual desire

d) Difficulty achieving orgasm

e) Increased fertility

58) Which of the following occurs in 35-59% of men with diabetes mellitus (DM)?

a) Priapism

b) Impotence

Page 55: Quiz Human Sexuality

Human Sexuality 14Mar2009

DO NOT DISTRIBUTE - 55 -

c) Premature ejaculation

d) Dry run orgasms

e) Increase sex drive

59) Diabetics with erectile dysfunction are more likely to develop which of the

following?

a) Retinopathy

b) Neuropathy

c) Leukopathy

d) Myopathy

e) A & B

60) Which of the following is usually recommended for patients who have received a

coronary artery bypass graft (CABG)?

a) Return to normal sexual activity

b) Increase sexual activity to strengthen the heart

c) Calm, non-athletic sexual activity

d) No sexual activity

61) Onset of erectile dysfunction in a man over 35 years old may be a warning sign of:

a) Neurological disorders

b) Liver dysfunction

c) Arterial disease

d) Spinal disease

e) Diabetes

62) Which of the following is usually recommended for patients who have had a

myocardial infarction with negative catheterization a week ago or four weeks after no

symptoms or EKG changes are noted?

a) Return to normal sexual activity

b) Increase sexual activity to strengthen the heart

c) Calm, non-athletic sexual activity

d) No sexual activity

63) Increased heart rate (HR) and blood pressure (BP) at maximum levels with orgasm

were seen to be less than with modest activity of a non-sexual nature.

a) True

b) False

64) Which of the following is the most likely barrier to sexual activity for obese and

anorexic patients?

a) Hypertension

b) Lethargy

c) Diabetes

d) GI complaints

e) Psychological

65) Performing in a female superior sexual position may have no apparent advantage for

a man with hypertension.

a) True

b) False

66) Patients with rheumatic diseases (osteoarthritis, rheumatoid arthritis) may have which

of the following sexual symptoms?

Page 56: Quiz Human Sexuality

Human Sexuality 14Mar2009

DO NOT DISTRIBUTE - 56 -

a) Excessive joint movement

b) Increased vaginal moisture

c) Fatigue

d) Priapism

e) Increased libido

67) Post-stroke mechanical problems such as weakness or spasticity may be overcome

with modified sexual techniques.

a) True

b) False

68) Sexual activity has been shown not to provoke seizure episodes in epileptic patients.

a) True

b) False

69) Patients with Alzheimer disease, especially females, are typically regarded as

“hypersexual.”

a) True

b) False

70) One in two men with Alzheimer disease will have erectile dysfunction.

a) True

b) False

71) Gay men may have an increased risk for Giardia lamblia, Entamoeba histolytica,

Shigella species, Campylobacter species, and Hepatitis A due to which of the following?

a) Oral-anal sexual contact

b) Anal-receptive; oral genital contact

c) Smoking; decreased frequency of screening

d) Increased HPV and HCV from anal-receptive

e) Subculture attitudes and pressure toward beauty

72) Lesbians may have an increased risk for certain types of cancer. Which of the

following risk factors is the least likely to be modified for these women to help prevent

cancer?

a) Smoking

b) Increased alcohol use

c) Fewer screening exams

d) Higher Body Mass Index (BMI)

e) Nulliparity

73) Lesbians may have a decreased risk for STDs as well as which of the following types

of cancer?

a) Breast cancer

b) Ovarian cancer

c) Cervical cancer

d) Endometrial cancer

e) Colon cancer

74) Gay men and women are at a higher risk for stroke, coronary artery disease (CAD),

and lung cancer due to which of the following risk factors?

a) Promiscuity

b) Identity crisis and psychological issues

c) Increased stress

Page 57: Quiz Human Sexuality

Human Sexuality 14Mar2009

DO NOT DISTRIBUTE - 57 -

d) Increase smoking

e) Fewer screening exams

75) The actual or perceived risk and associated fear of entering a homophobic and

potentially abusive medical system outweigh the perceived risk of illness.

a) True

b) False

76) The phrase “I’m with Dorothy” was used during WWII for military men meaning

they were:

a) Hiding from battle

b) Against gay males

c) Against lesbians

d) Under attack

e) Homosexual

77) A female that is used by a gay man to hide behind in social events to avoid suspicion

is termed a:

a) Butch

b) Dyke

c) Frottage

d) Beard

e) Fluffer

78) Terms used to indicate discomfort in the testicles when ejaculation is delayed (sexual

frustration) include blue balls and:

a) Hard on

b) Freeze frame

c) Frigidity

d) Tricking

e) Impotence

79) Butch is a term used to denote:

a) A gay male who is masculine

b) A gay male who is feminine

c) A lesbian who is masculine

d) A lesbian who is feminine

e) A straight male who is very strong

80) Cornholing is a term used to denote:

a) Oral sex

b) Vaginal sex

c) Anal sex

d) Ejaculation

81) The derogatory term “fag hag” refers to:

a) A women who befriends and socializes with gay men

b) A women who befriends and socializes with lesbians

c) A man who befriends and socializes with gay men

d) A man who befriends and socializes with lesbians

82) The terms felching, shrimping, snowballing, and rainbow kiss have which of the

following in common?

a) They involve oral sex

Page 58: Quiz Human Sexuality

Human Sexuality 14Mar2009

DO NOT DISTRIBUTE - 58 -

b) They involve anal sex

c) They involve vaginal sex

d) They involve semen

83) A hired member of a pornographic video crew whose job is to arouse the male

participants prior to filming is termed a:

a) Butch

b) Dyke

c) Frottage

d) Beard

e) Fluffer

84) Non-consensual rubbing against another person to achieve sexual arousal is called:

a) Felching

b) Frottage

c) Sodomy

d) Rimming

e) Onanism

85) The gay liberation movement started at a small Greenwich Village bar, The

Stonewall Inn, the day after Judy Garland’s funeral in what year?

a) 1828

b) 1917

c) 1939

d) 1969

e) 2008

86) Klismaphilia is the act of sexual play involving:

a) Feces

b) Plants

c) Enemas

d) Animals

e) Dead bodies

87) Necrophilia is the act of sex involving:

a) Feces

b) Plants

c) Enemas

d) Animals

e) Dead bodies

88) The biblical term for masturbation or “spilling the seed on the ground” is:

a) Felching

b) Frottage

c) Sodomy

d) Rimming

e) Onanism

89) The term queen is used to denote:

a) A gay male who is masculine

b) A gay male who is feminine

c) A lesbian who is masculine

d) A lesbian who is feminine

Page 59: Quiz Human Sexuality

Human Sexuality 14Mar2009

DO NOT DISTRIBUTE - 59 -

e) A straight male who is very strong

90) Sexual activity involving the tongue on the anus is termed:

a) Felching

b) Frottage

c) Sodomy

d) Rimming

e) Onanism

91) Scat (coprophilia) is sexual activity involving:

a) Feces

b) Plants

c) Enemas

d) Urine

e) Dead bodies

92) Different colored bandanas can be placed in a specific back pant pocket to indicate

sexual activity preference. Colored wrist-bands can also be used to indicate sexual

activity or the accomplishment of sexual acts.

a) True

b) False

93) Although there is little significance today, the phrase “left is right and right is wrong”

referred to homosexual preference denoted by:

a) Handkerchief pocket

b) Ear piercing

c) Wrist to wear a watch on

d) Shoe laces

e) Which foot to tap in the bathroom

94) Size queen is a term used to denote:

a) A gay male who chooses a partner based on how strong they are

b) A gay male who chooses a partner based on their penis size

c) A female who chooses a partner based on how strong they are

d) A female who chooses a partner based on their penis size

e) A lesbian who chooses a partner based on how strong she is

95) Transvestite refers to which of the following?

a) Cross-dressing for fun

b) Men who wear women’s clothes

c) Females who dress “in drag”

d) Drag queens

e) All of the above

96) Tricking is a term that denotes:

a) Dressing in drag

b) Having anal sex

c) Having oral sex

d) Selling oneself for sex

e) Lying about one’s sexual preference

97) The terms “water sport” and “golden shower” refer to sexual activity involving:

a) Feces

b) Plants

Page 60: Quiz Human Sexuality

Human Sexuality 14Mar2009

DO NOT DISTRIBUTE - 60 -

c) Enemas

d) Urine

e) Dead bodies

98) Studies show that individual counseling by physicians is the single most powerful

factor in persuading patients to change unsafe health behaviors.

a) True

b) False

99) Cancer of the testicles is the number one cancer in teenage boys and young men.

a) True

b) False

100) When putting on a condom, it is important to have the tip of the condom tight

against the glans penis.

a) True

b) False

101) The mons veneris is also known as the mons pubis.

a) True

b) False

102) In general, if a woman is nulliparous (has not had children) only the labia majora of

the vagina will be visible upon inspection in anatomical position.

a) True

b) False

103) The labia minora form a fourchette at what anatomical location?

a) Anterior

b) Posterior

c) Lateral

104) Which of the following is the strongest pelvic floor muscle and can cause

vaginismus if it spasms?

a) Iliococcygeus

b) Pubococcygeus

c) Levator ani

d) Puborectalis

105) Honeymoon cystitis is a type of bladder infection associated with:

a) Initial (virgin) sex

b) Frequent vaginal sex

c) Anal sex

d) Cranberry juice

e) Spermicidal lubricated condom

106) Which of the following uterine positions is the most common?

a) Anteverted

b) Midposition

c) Retroverted

107) Cryptochidism refers to a testicle that is:

a) Atrophied

b) Enlarged

c) Bleeding

d) Necrosing

Page 61: Quiz Human Sexuality

Human Sexuality 14Mar2009

DO NOT DISTRIBUTE - 61 -

e) Undescended

108) Which of the following is just beyond the urethral bulb and contributes about 30%

of the accessory fluid to the seminal ejaculate?

a) Prostate

b) Bulbourethral gland

c) Seminal vesicle

d) Epididymis

e) Seminiferous tubules

109) Which of the following produces secretions during sexual arousal (pre-ejaculate,

pre-come, or “pre-cum”)?

a) Prostate

b) Bulbourethral gland

c) Seminal vesicle

d) Epididymis

e) Seminiferous tubules

110) Kegel exercises are designed to strengthen which of the following muscles?

a) Iliococcygeus

b) Pubococcygeus

c) Levator ani

d) Puborectalis

111) Which of the following is NOT a phase of sexual response?

a) Excitement phase

b) Plateau phase

c) Orgasmic phase

d) Ejaculation phase

e) Resolution phase

112) If penile entry is attempted in a sexually unaroused female, the flaccid labia minora

may be carried into the vaginal opening causing discomfort and possibly trauma.

a) True

b) False

113) Pain in late arousal for a female could indicate which of the following?

a) HIV

b) Clitoris trauma

c) Bartholin retention cyst

d) Menopause

e) Perineal tear

114) Which of the following arteries supplies the erectile tissues as well as the helicine

arteries of the penile corporal bodies?

a) Internal pudendal artery

b) Perineal artery

c) Cavernosal artery

d) Bulbourethral artery

e) Dorsal artery of the penis

115) A drug that inhibits PDE 5, which degenerates cGMP, thus increasing cGMP and

promoting smooth muscle relation would cause which of the following?

a) Sexual dysfunction

Page 62: Quiz Human Sexuality

Human Sexuality 14Mar2009

DO NOT DISTRIBUTE - 62 -

b) Flaccidity

c) Erection

21 – Human Immunodeficiency Virus (HIV)

1) Within the M (major) group of HIV-1, there are 10 subtypes including clades B and C.

Which of the following is NOT included in clade B?

a) Americas

b) Japan

c) Australia

d) South Africa

e) Caribbean

2) HAART (Highly Active Antiretroviral Therapy) usually includes which of the

following?

a) Nucleoside reverse transcriptase inhibitor (NRTI), non-nucleoside reverse

transcriptase inhibitor (NNRTI), and transcription inhibitor (TI)

b) Nucleoside reverse transcriptase inhibitor (NRTI), transcription inhibitor (TI),

and protease inhibitor (PI)

c) Nucleoside reverse transcriptase inhibitor (NRTI), non-nucleoside reverse

transcriptase inhibitor (NNRTI), and protease inhibitor (PI)

d) Non-nucleoside reverse transcriptase inhibitor (NNRTI), transcription inhibitor

(TI), and protease inhibitor (PI)

3) Which of the following has the highest risk of HIV transmission?

a) Blood transmission

b) Blood product transmission

c) Sexual transmission

d) Vertical transmission (maternal/fetal)

4) Which of the following drugs has been shown to decrease transmission rate to 8.3% if

taken after the 14th week of gestation?

a) AZT, Zidovudine (Retrovir)

b) 3TC, Lamivudine (Epivir)

c) ddC, Zalcitabine (Havid)

d) ddI, Didanosine (Videx)

e) Delaverdine (Rescriptase)

5) Starting in 2007, the Center for Disease Control (CDC) recommends:

a) Asking high risk patients about HIV

b) Asking all patients about HIV

c) Testing high risk patients for HIV

d) Testing all patients for HIV

e) Not allowing patients to “opt out” of testing

6) In Pennsylvania (PA), All HIV positive antibody tests and positive viral loads and low

CD4 counts are reportable to the PA Health Department.

a) True

b) False

7) Pennsylvania Act 148 mandates all of the following for HIV testing EXCEPT:

a) Consent

b) Pretest counseling

Page 63: Quiz Human Sexuality

Human Sexuality 14Mar2009

DO NOT DISTRIBUTE - 63 -

c) Confirmatory test

d) Notification of sexual partners

e) Posttest counseling

8) An enzyme-linked immunosorbent assay (ELISA) test is used as preliminary HIV

screening with a Western Blot being used for confirmation.

a) True

b) False

9) The HIV window period between initial infection and onset of detectable antibodies is

typically how long?

a) Less than one month

b) One to three months

c) Three to six months

d) Six months to a year

e) At least a year

10) The OraQuick Advance by OraSure detects antibodies to HIV-1 and HIV-2 in about

20 minutes and can use all of the following types of fluid EXCEPT:

a) Oral fluid

b) Semen

c) Whole blood

d) Blood plasma

11) Patients infected with HIV can receive live vaccinations.

a) True

b) False

12) Testing positive for HIV requires ELISA and Western Blot, or a positive viral load

with acute retroviral infection symptoms in primary HIV infection.

a) True

b) False

13) With a positive HIV antibody serology, acquired immune deficiency syndrome

(AIDS) is diagnosed with an opportunistic infection (OI) or a CD4 count less than:

a) 10,000 cells/mm^3

b) 1,000 cells/mm^3

c) 200 cells/mm^3

d) 50 cells/mm^3

e) 1 cell/mm^3

14) Which of the following is the most common symptom associated with Acute

Retroviral Syndrome (PHI)?

a) Fever

b) Lymphadenopathy

c) Pharyngitis

d) Rash

e) Myalgia

15) HIV treatment with HAART should not begin unless a patient is symptomatic with an

opportunistic infection (OI) or with PHI.

a) True

b) False

Match the CD4 cell/mm^3 count with the prophylaxis treatment:

Page 64: Quiz Human Sexuality

Human Sexuality 14Mar2009

DO NOT DISTRIBUTE - 64 -

16) Toxoplasma and thrush a) 200 CD4 cells/mm^3

17) Pneumocyctis Pneumonia (PCP) b) 100 CD4 cells/mm^3

18) Disseminated Mycobacterium Avium Complex (MAC) c) 50 CD4 cells/mm^3

19) It is important to know the lowest CD4 level prior to HIV treatment, as when anti-

retroviral therapy is stopped the body will easily return to that CD4 level.

a) True

b) False

20) “Viral blips” or transient low level rises have been shown to be significant in

determining AIDS progression.

a) True

b) False

21) HIV viral eradication is possible with today’s drugs.

a) True

b) False

22) Viral reservoirs are the most common in which of the following locations?

a) CNS

b) Genital tract

c) GI associated lymphoid tissue (GALT)

d) Mucosa associated lymphoid tissue (MALT)

e) Spleen

23) Results of different tests, such as bDNA and PCR, are normalized so they are

equivalent for diagnosis.

a) True

b) False

24) What percentage of newly infected individuals have resistance to at least one drug

prior to initiating treatment for HIV?

a) 0.5%

b) 1%

c) 2%

d) 5%

e) 15%

25) A trofile (tropism test) uses M-topic R5 virus to show ____ infection and T-tropic X4

virus to show _____ infection.

a) Early; Terminal

b) Early; Early

c) Early; Late

d) Late; Early

e) Late; Terminal

26) Resistance testing requires samples of >1000 viral load copies and measures majority

variants (20% or more of the viral population). If a reported mutation came back as

M184V, how would this be interpreted?

a) The major amino acid was moved 184 positions

b) At amino acid 184, the normal type “V” has become type “M”

c) At amino acid 184, the normal type “M” has become type “V”

d) Normal type “V” amino acid has moved 184 positions to “M”

e) Normal type “M” amino acid has moved 184 positions to “V”

Page 65: Quiz Human Sexuality

Human Sexuality 14Mar2009

DO NOT DISTRIBUTE - 65 -

27) Which of the following HIV drug treatment sites affects cellular exiting of HIV?

a) Chemokine receptors

b) Reverse transcriptase

c) Integrase

d) Protease

e) Maturation

28) Which of the following is a side effect of antiretroviral drugs?

a) Wasting syndrome

b) Anemia

c) Dementia

d) Fat redistribution

e) Opportunistic disease

29) HIV treatment should be offered when CD4 counts are:

a) < 50 cells/mm^3

b) < 200 cells/mm^3

c) 200-350 cells/mm^3

d) > 500 cells/mm^3

e) 500-100,000 cells/mm^3

30) Adipose tissue accumulation seen with HAART therapy is mostly distributed around:

a) The brain

b) Viscera

c) The limbs

d) The abdomen

e) Breasts

31) A “Buffalo Hump” or “pseudo-Cushing Syndrome” seen with HAART therapy is

associated with fat accumulating around:

a) The brain

b) Viscera

c) The limbs

d) The abdomen

e) Breasts

32) Which of the following risk factors for HIV positive patients is seen significantly

more in men then women?

a) Insulin resistance

b) Cardiovascular disease

c) Decreased bone mineral deposits

d) Avascular necrosis

e) Anal cancer

33) Which of the following is the treatment of choice for Pneumocystis jiroveci (carinii)

as well as Toxoplasma gondii?

a) Sulfacetamide

b) Trimethoprim

c) Co-trimazole (TMP-SMX)

d) Phenobarbital

e) Clarithromycin (Biaxin)

34) Which of the following is a drug used for Mycobacterium avium complex?

Page 66: Quiz Human Sexuality

Human Sexuality 14Mar2009

DO NOT DISTRIBUTE - 66 -

a) Sulfacetamide

b) Trimethoprim

c) Co-trimazole (TMP-SMX)

d) Phenobarbital

e) Clarithromycin (Biaxin)

35) vircoTYPE HIV-1 by Vircolab is an HIV-1 analysis system that combines which of

the following?

a) Genotype information

b) Phenotype information

c) Clinical information

d) All of the above

36) A patient test postive for HIV via a serology test. Their CD4 count is at 10,000

copies. Using the CD4 percentage calculation, this patient would be diagnosed with

AIDS if their CD4 count dropped below:

a) 50 copies

b) 140 copies

c) 200 copies

d) 1,400 copies

e) 2,000 copies

37) Which of the following was NOT included in the first successful HAART treatment,

Merck 035 study?

a) AZT

b) ZDV

c) 3TC

d) Indinavir

38) Which of the following PDE-5 inhibitors has the longest half life?

a) Sildenafil (Viagra)

b) Vardenasil (Levitra)

c) Vardenasil (Lexiva)

d) Vardenasil (Lexepro)

e) Tadalafil (Cialis)

22 – Rape Crisis & Sexual Abuse

1) When children are sexually abused, they know their abuser ____ of the time.

a) 15%

b) 25%

c) 50%

d) 75%

e) 85%

2) Preferential abusers (e.g. pedophiles) are the more common type of child sexual

abusers.

a) True

b) False, situational abusers are more common

3) How many women will be sexually assaulted in their lifetime?

a) 1 in 5

b) 1 in 4

Page 67: Quiz Human Sexuality

Human Sexuality 14Mar2009

DO NOT DISTRIBUTE - 67 -

c) 1 in 3

d) 1 in 2

e) Nearly all

4) How many men will be sexually assaulted in their lifetime?

a) 1 in 8

b) 1 in 6

c) 1 in 4

d) 1 in 2

e) Nearly all

5) Sexual assault is the most under-reported crime and is reported about ____ of the time.

a) 20%

b) 40%

c) 60%

d) 80%

6) Which of the following is NOT a reason why the highest risk period for sexual abuse

is between ages 16 and 24?

a) Alcohol use

b) Getting driver’s license

c) Starting to date

d) More freedom from parents

e) Unsure of what they want sexually

7) What is the age of consent for statutory sexual assault in the state of Pennsylvania?

a) 13

b) 14

c) 15

d) 16

e) 17

f) 18

8) Regarding meaningful consent, a person who is drunk or high on any other substances

cannot consent to sex.

a) True

b) False, the person must be legally intoxicated per state law

9) Which of the following is the most common date rape drug and is seen in over 70% of

rapes on college campuses?

a) Paracetamol (acetaminophen) and diphenhydramine (Tylenol PM)

b) Gamma-Hydroxybutyric acid (GHB)

c) Flunitrazepam (Rohypnol)

d) Ketamine

e) Midazolam

f) Alcohol

10) Which of the following is NOT true of most date rape drugs?

a) Translucent

b) Odorless

c) Non-polar

d) Tasteless

e) Have amnesia-like effects

Page 68: Quiz Human Sexuality

Human Sexuality 14Mar2009

DO NOT DISTRIBUTE - 68 -

11) Which of the following reduces the gag-reflex and thus can increase the risk of

aspiration and/or choking?

a) Paracetamol (acetaminophen) and diphenhydramine (Tylenol PM)

b) Gamma-Hydroxybutyric acid (GHB)

c) Flunitrazepam (Rohypnol)

d) Ketamine

e) Midazolam

f) Alcohol

12) Which drug is considered ten times stronger than Valium, has associated memory

loss for 8-12 hours, and can last for 2-3 days?

a) Midazolam

b) Paracetamol (acetaminophen) and diphenhydramine (Tylenol PM)

c) Gamma-Hydroxybutyric acid (GHB)

d) Flunitrazepam (Rohypnol)

e) Ketamine

13) Which of the following drugs is associated with hallucinations and flashbacks?

a) Midazolam

b) Paracetamol (acetaminophen) and diphenhydramine (Tylenol PM)

c) Gamma-Hydroxybutyric acid (GHB)

d) Flunitrazepam (Rohypnol)

e) Ketamine

14) What percentage of sexual assault reports are false?

a) 2%

b) 10%

c) 15%

d) 20%

e) 50%

15) Which of the following is NOT a common symptom of post-traumatic stress disorder

(PTSD)?

a) Nightmares

b) Depression

c) Jumpiness

d) Euphoria

e) Flashbacks

16) About how long can the disorganization phase of rape-trauma syndrome (RTS) last?

a) 24-hours

b) Four days

c) Two weeks

d) Two months

e) Four years

17) The reorganization phase of RTS can last up to how long?

a) 24-hours

b) Four days

c) Two weeks

d) Two months

e) Four years

Page 69: Quiz Human Sexuality

Human Sexuality 14Mar2009

DO NOT DISTRIBUTE - 69 -

18) Which of the following is the most important factor that influences recovery after

RTS?

a) Relationship with offender

b) Degree of violence

c) Social influences

d) Cultural influences

e) Immediate personal contacts after the assault

19) In the United States, there is a rape every:

a) Month

b) Day

c) Hour

d) 30-minutes

e) 3-minutes

20) Forensic nurse examiners are registered nurses who are specially trained to provide

comprehensive care to which of the following?

a) Victims of violence

b) Victims of sexual assault

c) Child abuse victims

d) Elder abuse victims

e) All of the above

21) During a sexual assault genital exam, a speculum should not be used if the patient has

not started their menstrual cycle (menarche).

a) True

b) False, a speculum exam should always done

22) Which of the following locations is the most common for genital injury during sexual

assault?

a) Hymen (ecchymosis)

b) Fossa navicularis (tears)

c) Anus (tears)

d) Posterior forchette (tear)

e) Vagina (ecchymosis)

23) An Emergency Room physician would not examine a sexual assault patient’s genital

area unless requested by the forensic nurse examiner (FNE).

a) True

b) False

24) Toluidine Blue Dye, which adheres to nucleated squamae in the deeper layers of the

epidermis, should be applied:

a) To the assault victim’s hands before evidence collection

b) To the assault victim’s hands after evidence collection

c) To the assault victim’s genital area before evidence collection

d) To the assault victim’s genital area after evidence collection

25) Which of the following is tested via blood and not culture?

a) Gonorrhea

b) Chlamydia

c) Syphilis

d) Trichamonis

Page 70: Quiz Human Sexuality

Human Sexuality 14Mar2009

DO NOT DISTRIBUTE - 70 -

26) Blood samples taken for a suspected drug facilitated sexual assault should be drawn

(gray top blood tube) up to how many hours after an incident?

a) 6

b) 12

c) 24

d) 48

e) 96

27) Date rape drugs may remain detectable in the urine up to how many hours?

a) 6

b) 12

c) 24

d) 48

e) 96

Match the medication with the reason for treatment:

28) Chlamydia a) Ceftriaxone (Rocephin)

29) Pregnancy b) Azithromycin (Zithromax)

30) Gonorrhea c) Levonorgestrel (Plan B)

31) In what situation(s) can physician-patient confidentiality be broken?

a) Never (HIPAA laws)

b) If the patient plans to harm them self

c) If the patient plans to harm others

d) If a minor is being abused

e) B, C, & D

James Lamberg

Page 71: Quiz Human Sexuality

Human Sexuality 14Mar2009

DO NOT DISTRIBUTE - 71 -

AnswerKey

HS #1

1) E

2.1) B

2.2) D

2.3) A

2.4) C

3) C

4.1) A

4.2) C

4.3) D

5.1) A

5.2) E

5.3) A

5.4) C

5.5) D

6) E

7) A

8.1) B

8.2) A

8.3) A

8.4) B

8.5) A

8.6) A

8.7) A

8.8) B

9.1) D

9.2) C

9.3) C

9.4) E

9.5) D

9.6) A

HS #2

1) E

2.1) C

2.2) D

3) B

4.1) C

4.2) A

5.1) A

5.2) A

5.3) A

5.4) B

5.5) A

5.6) B

6.1) C

6.2) B

6.3) D

HS #3

1.1) A

1.2) E

1.3) D

2) C

3) E

4.1) A

4.2) B

5) B

6.1) A

6.2) D

7.1) C

7.2) A

8) E

9) E

10) A

11.1) D

11.2) E

11.3) A

12.1) A

12.2) A

12.3) B

12.4) B

12.5) B

12.6) B

13.1) A

13.2) C

13.3) A

13.4) B

13.5) A

13.6) E

13.7) D

13.8) B

13.9) B

13.10) A

13.11) E

13.12) D

13.13) C

13.14) B

13.15) A

13.16) E

13.17) E

13.18) A

13.19) E

13.20) A

13.21) C

13.22) B

13.23) D

13.24) A

13.25) D

13.26) B

14.1) D

14.2) C

HS #4

1.1) D

1.2) C

2) C

3.1) B

3.2) E

4) D

5.1) A

5.2) C

6) E

7) E

8.1) C

8.2) B

9.1) A

9.2) B

10) A

11) C

12.1) D

12.2) E

12.3) A

13.1) D

13.2) E

14.1) B

14.2) A

14.3) A

14.4) B

14.5) B

14.6) B

14.7) A

15.1) C

15.2) C

15.3) A

15.4) A

16.1) E

16.2) C

17) A

18) B

HS #5

1) D

2) A

3) B

4) C

5.1) A

5.2) D

6) C

7) C

8) C

9) A

10) D

11) B

12) C

13) B

14.1) A

14.2) B

14.3) A

14.4) A

14.5) A

15.1) C

15.2) A

16.1) A

16.2) E

HS #6

1.1) D

1.2) D

2) E

3.1) C

3.2) B

3.3) A

3.4) A

4.1) A

4.2) A

4.3) A

4.4) B

5.1) C

5.2) A

5.3) C

5.4) E

6) A

HS #9

1.1) A

1.2) A

2) D

3.1) A

3.2) A

4) E

5) C

6) A

7) E

8.1) A

8.2) B

8.3) D

9) A

10.1) A

10.2) D

11.1) B

11.2) A

11.3) B

11.4) B

11.5) B

11.6) B

11.7) A

11.8) B

11.9) B

12.1) C

12.2) D

HS #10

1) B

2.1) E

2.2) B

3.1) C

3.2) B

4) D

5.1) A

5.2) A

5.3) A

6) A

7) C

8) C

9.1) A

9.2) B

10) E

11.1) B

Page 72: Quiz Human Sexuality

Human Sexuality 14Mar2009

DO NOT DISTRIBUTE - 72 -

11.2) A

12) D

13.1) E

13.2) A

14) B

15.1) B

15.2) A

15.3) A

15.4) A

15.5) B

15.6) A

16.1) B

16.2) B

16.3) D

HS #11

1) D

2) E

3.1) E

3.2) D

4.1) E

4.2) B

5) B

6) C

7.1) A

7.2) B

7.3) A

7.4) A

7.5) A

7.6) B

7.7) B

7.8) A

8) D

HS #12

1) E

2.1) C

2.2) D

3) B

4) B

5) E

6) E

7) D

8) A

9) D

10) C

11) D

12) B

13) C

14.1) A

14.2) A

14.3) B

14.4) A

14.5) A

14.6) B

14.7) B

15.1) B

15.2) C

15.3) A

16.1) A

16.2) A

16.3) A

HS #13

1) C

2) B

3) B

4.1) B

4.2) C

5) A

6) A

7) B

8) E

9) C

10) C

11.1) A

11.2) B

12) C

13) C

14) C

15) D

16) B

17) A

18) A

19.1) A

19.2) B

19.3) B

19.4) B

19.5) A

19.6) A

20.1) C

20.2) A

21.1) A

21.2) B

22) A

23) A

HS #14

1) A

2) D

3) A

4) D

5) A

6) A

7) C

8) A

9) D

10) E

11) A

12.1) A

12.2) A

12.3) A

12.4) A

12.5) B

12.6) B

12.7) B

12.8) B

13.1) D

13.2) A

HS #15

1) D

2.1) A

2.2) B

2.3) D

3) C

4) E

5) E

6) A

7.1) B

7.2) A

8) A

9.1) A

9.2) D

10) A

11) A

12.1) B

12.2) B

12.3) B

12.4) B

12.5) A

12.6) A

12.7) A

13.1) A

13.2) D

14.1) A

14.2) B

HS #16

1) A

2.1) C

2.2) E

2.3) A

3) E

4.1) L

4.2) K

4.3) J

4.4) I

4.5) H

4.6) G

4.7) F

4.8) E

4.9) D

4.10) C

4.11) B

4.12) A

5.1) B

5.2) A

5.3) E

6.1) C

6.2) D

6.3) E

6.4) B

6.5) A

7) E

8) D

9) F

10) C

11) B

12) C

13) C

14) B

15) B

16.1) A

16.2) B

16.3) B

16.4) B

16.5) A

16.6) A

16.7) B

17.1) C

17.2) D

17.3) B

17.4) B

HS #18

1) A

2) A

3) C

4) A

5) A

6) A

7) C

8) A

9) E

10.1) A

10.2) A

10.3) B

10.4) B

10.5) B

10.6) A

11) D

HS #19

1) B

2) E

3.1) B

3.2) B

3.3) A

3.4) B

3.5) A

3.6) A

4.1) B

4.2) A

HS #20

1) E

2) E

3) B

4) B

Page 73: Quiz Human Sexuality

Human Sexuality 14Mar2009

DO NOT DISTRIBUTE - 73 -

5) D

6) C

7) E

8) A

9) E

10) E

11) D

12) E

13) D

14) D

15) B

16) A

17) E

18) A

19) C

20) B

21) E

22) C

23) C

24) C

25) D

26) E

27) A

28) D

29) B

30) C

31) E

32) F

33) E

34) D

35) C

36) A

37) C

38) E

39) C

40) D

41) C

42) F

43) A

44) E

45) F

46) D

47) C

48) A

49) E

50) A

51) A

52) A

53) C

54) D

55) A

56) C

57) D

58) B

59) E

60) D

61) C

62) A

63) A

64) E

65) A

66) C

67) A

68) B

69) B

70) A

71) A

72) E

73) C

74) D

75) A

76) E

77) D

78) B

79) C

80) C

81) A

82) D

83) E

84) B

85) D

86) C

87) E

88) E

89) B

90) D

91) A

92) A

93) B

94) B

95) E

96) D

97) D

98) A

99) A

100) B

101) A

102) A

103) B

104) C

105) B

106) A

107) E

108) A

109) B

110) B

111) D

112) A

113) C

114) E

115) C

HS #21

1) D

2) C

3) C

4) A

5) D

6) A

7) D

8) A

9) C

10) B

11) B

12) A

13) C

14) A

15) B

16) B

17) A

18) C

19) A

20) B

21) B

22) C

23) B

24) E

25) C

26) C

27) E

28) D

29) C

30) B

31) E

32) E

33) C

34) E

35) D

36) D

37) A

38) E

HS #22

1) E

2) B

3) C

4) B

5) A

6) E

7) D

8) A

9) F

10) C

11) B

12) C

13) E

14) A

15) D

16) D

17) E

18) E

19) E

20) E

21) A

22) D

23) A

24) D

25) C

26) C

27) E

28) B

29) C

30) A

31) E